Вы находитесь на странице: 1из 77

AUGUST 18th

MAJOR RULES (90.39%)


WON'T BE TESTED ON THE CODE OF JUDICIAL CONDUCT
memorize rules
memorize specific cases
EXAM: MC same across all PR classes.
look at Moore and Vestrand in the exam bank
EXAM tip: state GR and exceptions IF they apply to the facts
EXAM TIP: look at dates to see if lawyer's services were used in furtherance of the crime or fraud
MC question: Do the MC at back of the book.
EXAM TIP: “It is professional misconduct for a lawyer to violate or attempt to violate the Rules of Professional
Conduct, knowingly assist or induce another to do so, or do so through the acts of another. It is a violation to
violate the rules.” Easy points on the exam for this + harmed reputation on both essays

2 essays 70 and 115


MC 25*3 = 75pts
lots of opportunities for points
260 pts
165 minutes

Sat Aug 18 9am – 11:45am


9am start MC
9:48 end MC start essay #1
10:32 end essay #1, start big essay

Overview of the ABA Model Rules of Professional Conduct


1969 – ABA Code of Professional Responsibility and was adopted in several states.
Problems with 1969 code : standards too broad and too general. Watergate = 40 lawyers including President.
Their defense: code of prof responsbility failed to give them specific or clear notice that things like conspiracy,
burglar, and espionage were unetheic and improer conduct for an attny. Model Rules of Professional Conduct
came about. In the wake of Enron, WorldCom and HealthSouth it was clear rules required further revision b/c
lawyers argued that their duty of confidentiality prevented them from acting. ABA launched “Ethics 2000.”
ABA Rules are “aspirational” only = not binding. Michigan Rules of Professional Conduct ARE binding. See
“West's Rules of Court” and see state bar's website

Rule 1.0: Terminology


“Firm:” Lawyers in a law partnership, professional corporation, sole proprietorship or other
association authorized to practice law including legal services organizations and legal departments of
corporations and other organizations.

“Informed Consent” agreement by a person to a proposed course of conduct after the lawyer has
communicated adequate info and explaination about the material risks AND reasonably available
alternatives to the proposed course of conduct.

K, Kingly – a person's K may be inferred from circumstances.

Reasonable: Conduct of a reasonably prudent and competent lawyer.


Reasonably should know: lawyer of reasonable prudence and competence would ascertain the matter in
question.

Screened: isolation of a lawyer from any participation in a matter through tiely imposition of procs
within firm that are reasonably adequeate under the cirucmstances to protect info that the isolated
lawyer is obligated to protect.

Substantion: a mater of clear and weighty importance.

Tribunal: a court, arbitrator in binding arbitration proceeding, legislative body admin agency or other
body acting in an adjudicative capacity.

Harmed reputation: If a lawyer violates the rules, he will harm his reputation

Rule 8.4 Misconduct


It is professional misconduct for a lawyer to do any of the following:
● violate or attempt to violate the Rules
● knowingly assist or induce another to violate the rules
● use someone else to violate the Rules
● commit a criminal act
traditionally involving “moral turpitude” but now offenses involving violence, dishonesty,
breach of trust, or serious interference with the admin of justice. A pattern of repeated
offenses, even if minor, can indicate indifference to legal obligation.
● engage in conduct involving dishonesty, fraud, deceit, misrepresentation, or is prejudicial to the
admin of justice
in course of representing a client, knowingly manifest by words or conduct bias or prejudice
based on race, religion, etc., that is outside legitimate advocacy (bad for admin of justice)
if a lawyer holds public office and abuses it
if a lawyer holds a position of private trust and abuses it (director or manager of a corp,
trustee, executor, administrator, guardian, agent, officer)
● state or imply ability to improperly influence a govt agency or official to achieve results by
means that violate the Rules
● knowingly assist a judge or judicial officer in conduct that violates CJC or other law

Rule 1.7 Conflict of Interest w/Current Clients


A lawyer shall not represent a client if there is a conflict of interest. A conflict of interest exists if any
of the following is true:
● the representation of one client is directly adverse to another client
● there is a significant risk that representing one client will be materially limited by
responsibilities to another client, former client, third person, or the personal interests of the
lawyer
critical question is the likelihood the difference in interests will eventuate and if it does
will it materially interfere?
In estate planning situations lawyer should clarify his relationship to the parties
involved.
A lawyer may represent a client even if conflict exists IF:
● lawyer reasonably believes he can still provide competent and diligent representation AND
representation not prohibited by law AND two clients aren't in direct conflict AND each
affected client gives informed consent confirmed in writing.
Law: some states provide that same lawyer cannot represent more than one D in a capital
case, and under fed crim statutes certain representations by former govt lawyers are
prohibited despite informed consent of former client.

“confirmed in writing” if it is not feasible to obtain at time person gives informed consent, then the
lawyer must obtain it within a reasonable time thereafter. Writing includes electronic transmission.
Writing required to impress upon clients the seriousness and avoid disputes or ambiguities.

“informed consent” agreement to a proposed course of conduct after the lawyer has explained the
material risks and reasonably available alternatives to the proposed course of conduct. Information
required may vary

If conflict exists before rep undertaken, the rep. Must be declined UNLESS informed consent obtained
ignorance of conflicts is no excuse, lawyer must adopt reasonable measures to identify conflicts and
resolve them.
1. Identify the client(s)
2. determine whether conflict exists
3. decide whether consentable (some aren't). Depends on whether the interess of clients will be
adequately protected.
4. consult with clients and obtain their informed consent confirmed in writing.

unforeseeable developments, such as changes in corp or organizational affiliations happens, lawyer has
option to withdraw but must seek court approval where necessary and take steps to minimize harm to
clients and protect confidences.

Under some situations consent to future conflicts is allowable. Depends on experience of the client and
other facts. However, advance consent cannot be effective if circumstances that materialize are non-
consentable.

Simultaneous rep of clients in unrelated matters whose interests are only economically adverse is not
normally a conflict of interest.

Potential for conflict in representing multiple crim defendants is so great normally lawyer should
decline.

Risks if common representation fails including additional costs and delays. Also, prevailing rule is that
between commonly represented clients, the attny-client priv does not attach.

A lawyer who represents a corp doesn't necessarily represent constituent or affiliated orgs such as a
parent or subsidiary.

Corp lawyer who is also a member of its board may end up in conflict and may need to give up one
role.

When lawyer represents a class of P's or D's in a class-action suit, unnamed members of class are NOT
ordinarily considered clients.

Negotiations between a seller and buyer are directly adverse.

If lawyers of adverse clients are closely related by blood or marriage, then informed consent is
necessary.

If lawyer is paid by somebody other than client, lawyer must make sure his duty of loyalty and
independent judgment to the client are not undermined. If accepting the payment poses a significant
risk of materially limiting rep (happens in co-client situations) then lawyer must determine if conflict is
consentable before he can accept the representation.

LAXTON's SLIDES ON CONFLICT OF INTEREST


What is a conflict of interest? Anything that interferes with:
1) Your loyalty to a client; OR
2) Your exercise of independent professional judgment on behalf of a client.
It’s something that weakens your actions for and on behalf of a client.

Potential Sources of a Conflict:


1. The lawyer’s own interest.
2. A third party.
3. Other clients or former clients.

Screen for a Conflict of Interest. Before you obtain any facts, determine the name of the party or
parties with whom the prospective client has a dispute. You may also have a conflict with potential
witnesses that are current or former clients.

Critical Question: can the same lawyer can serve competing interests loyally? Clients are entitled to a
lawyer’s undivided loyalty and zealousness. No man can equally serve two masters.

A concurrent conflict exists where:


1) the representation of one client will be directly adverse to (against) another client,
2) OR, there is a significant risk that the representation of one or more clients will be materially
limited by the lawyer’s responsibilities to another client, a former client, a third person or by a
personal interest of the lawyer.

Loyalty prohibits undertaking representation directly against a current client, even where the matters
are wholly unrelated.
Hypo: What if lawyer was representing you in a divorce case and then you are sued in a
personal injury case and YOUR lawyer was representing the plaintiff against YOU!?!

Significant Risk of Material Limitation – 1.7(a)(2):


Even in the absence of direct adverseness, a conflict also exists if there is a significant risk that a
lawyer’s ability to represent a client will be materially limited by the lawyer’s other responsibilities or
interests.
Example: The lawyer is asked to represent several persons in a joint venture. (Can the lawyer
really advocate all possible interests/positions with equal loyalty)?

Example: Co-plaintiffs or co-defendants


If a concurrent conflict is present in order to determine whether the concurrent representation is
permissible, an analysis is required. The conflict can be remedied by client consent or, whether the
conflict is so direct and intense, that regardless of client consent, the concurrent representation would
be impermissible.

The test has four prongs and ALL four prongs must be satisfied in order that the concurrent
representation be permissible. Where just one prong can not be satisfied, the conflict is impermissible
and it would be unethical for the lawyer to proceed on behalf of the competing interests.

Four Prong Test:


1. It must be reasonable to conclude that the lawyer can represent both clients competently and
diligently (with equal loyalty);
2. The concurrent representation must not be prohibited by law;
3. The concurrent representation cannot involve both sides of a claim in the same case; and,
4. Even if prongs (1) through (3) are satisfied, the representation is still impermissible unless each
affected client gives informed consent, confirmed in writing.

Prongs (1) – (3) must be satisfied before you can even seek consent!

Consent must be informed and in writing. MPRE trick!

Informed Consent: Rule 1.0(e) defines informed consent. In order to qualify as an informed consent
each affected client must be made aware of the relevant circumstances; and of the material and
reasonably foreseeable ways that the conflict could have adverse effects on the interests of that client.

Informed consent must include disclosure of the possible effects of the multiple representation on:
1) Loyalty,
2) Confidentiality, and:
3) The Attorney/Client Privilege.

The burden is on the attorney to initiate consent.

The attorney must be able to demonstrate that the client was of sufficient capacity and sophistication to
understand the implications of the concurrent representation.

Confirmed In Writing:
The confirmed in writing requirement is defined in Rule 1.0(b) and (n). According to these provisions,
an informed consent must be: Signed by the client; OR signed by the lawyer following an oral consent
and promptly transmitted to the client in confirmation of the consent. (If prompt transmission is
impossible, it must be transmitted within a reasonable time).

Whether a conflict is consentable depends on the circumstances.


Example: You can’t represent multiple parties to a negotiation whose interests are fundamentally
antagonistic to one another although you can engage in multiple representation where the clients are
generally aligned in interest.

Ordinarily, you must withdraw from the representation of all of the clients if the common
representation becomes impermissible.
If informed consent requires disclosures of another client under MR 1.6 then the lawyer cannot obtain
informed consent and cannot take the case.

Client consent can be revoked at any time!

Objective Standard: This Rule applies an objective as opposed to subjective standard: You must have
an actual belief that the representation will not adversely affect the relationship with the other client;
and, the belief must be reasonable. (A reasonable attorney in the same position must come to the same
conclusion). In other words, where a reasonable attorney could not conclude that you could represent
both clients without adverse affects to one of the clients, a direct conflict exists which can not be cured
through client waiver or consent.

Absolute Ban on Direct Conflicts


The absolute prohibition against direct conflicts exists to ward against two main dangers:
1. that confidential information will leak from one client to the other; and,
2. that clients and the public at large will be disturbed by a lawyer “playing both sides of the
street.”

Prejudice is Presumed
Prejudice is presumed when a lawyer actively represents clients with conflicting interests. Cuyler v
Sullivan, 446 US 335 (1980). The resultant appearance of impropriety directly undermines the
integrity of the legal profession.

Good Faith is No Defense


A lawyer’s claim that his conduct in representing both clients was not willful, will not succeed. The
defense of good faith is not available.

Need good screening procedures to protect the public’s confidence in the legal system.

Example Cases
G.A. v Peter Betrus (1995): Betrus accepted the representation of H, W and children in claims arising from
an auto accident involving a train. H was driving at the time of the collision. During discovery, it was
established that H may be contributorily negligent.

G.A. v Gregory Justis (1995): Justis’ client was charged with Possession of Cocaine. In exchange for
leniency, the client agreed to give grand jury testimony concerning his knowledge of certain drug trafficking
activity. While on the stand, the client implicated his good friend in the illegal activity, resulting in criminal
charges against the friend. Knowing no other lawyer, the newly charged individual asked Justis to represent
him, also. Justis accepted the case.

G.A. v Timothy Crawford (1994)


Four young men were cruising Woodward Avenue when another vehicle pulled up beside them. An
altercation broke out and the front seat passenger in the car carrying the four boys shot and killed a person in
the other car. All four boys were charged with first degree murder. The driver of that vehicle and one of the
boys in the backseat, a 14 year old, were cousins. The mothers of the two cousins hired Timothy Crawford to
defend their sons. Crawford netted a double fee. The cousins? How did they fare? While one cousin (the
driver) sat in his jail cell, Crawford negotiated a plea agreement on behalf of the other cousin (the 14 year old
back seat passenger). In exchange for testimony against his cousin and the shooter, the 14 year old would
receive immunity. Lesson: Never, ever represent co-defendants in a criminal case! Equal loyalty to both is
virtually impossible!
The representation of co-defendants/co-plaintiffs in civil cases is much more common. Nevertheless,
you must always be vigilant for a conflict!

When to Avoid Multiple Clients:


Substantial discrepancy in one client’s testimony vs. another client’s testimony.
Incompatibility of positions either in relationship to an opposing party or to each other.
Substantially different possibilities of outcome.
G.A. v David Klein (1994): Klein was the attorney for an iron workers union and its fringe benefit trusts,
including the “SUB Fund.” After the union’s business manager was fired from his position, Klein agreed to
represent him in claims against the SUB Fund.

G.A. v Lawrence Greene (1995): Greene was the attorney for a client jailed on drug-related crimes.
During the representation, Greene agreed to represent the client’s wife in a divorce action against his client.
Hmmm…

Matters Unrelated!
The Greene case is a perfect example of the impropriety of concurrent representation even where the
matters are wholly unrelated! One matter was a criminal case and the other a civil case and the two
cases had no relationship to one another. The critical fact is that the new matter (the divorce) was
directly against a current client!
Example: G.A. v Moceri (1993): Moceri continued to represent the plaintiff in a PI case after
he discovered that the insurance carrier for the defendant was a client of the firm. The new
matter was against a current client!

Future Conflict Waivers


Whether waivers of future conflicts are proper depends on the following:
● Is the conflict waiveable (does it satisfy all four of the criteria of 1.7(b))?
● How well did the client understand the material risks of such a waiver?
● How sophisticated/experienced was the client?
● Was the client independently represented in giving the waiver?
● Is the waiver narrow or broad? (Generally, open ended waivers are much less likely to be
determined as valid/effective waivers – not reasonably likely that client will know of material
risks at the time).

Chinese Walls
A non-waiveable conflict is not made waiveable by a lawyer’s action in “walling off” the case from the
rest of the firm, unless the rules so permit. Chinese Walls or “screens” are only permitted in the case of
former government lawyers now in private practice (Model Rule 1.11). They do not apply to private
sector lawyers changing firms.

Close Family Relationships Between Counsel


If you and opposing counsel are closely related by blood or marriage (think back to “Class Action”),
there may be significant risk that:
1) client confidences will be revealed; and,
2) that the relationship will interfere with your loyalty and independent professional judgment.

Each client is entitled to know of the relationship for their consideration in determining whether to
retain you and their informed consent to the representation is required.
Example.: Cooley student whose brother is the Chief Prosecutor in the county in which he wants to do
criminal defense.

Note: The disqualification caused by a close family relationship is personal. Ordinarily it is not
imputed to other members of the firm with whom the lawyer is associated.

Positional Conflicts
The taking of inconsistent legal positions on behalf of different clients.
Example: Arguing on behalf of one client that polygraph evidence is scientifically reliable and arguing
on behalf of another client in a different case that this kind of evidence is unreliable.

GR: Ordinarily, you may take inconsistent legal positions in different tribunals at different times on
behalf of different clients.
Exception: Where there is a significant risk that your legal position on behalf of one client will
materially limit your effectiveness in representing another client.

Example: When a decision on behalf of one client will create a precedent likely to seriously
weaken another client’s case.

Factors for Determining Whether a Positional Conflict Exists


● Whether the common issues are substantive or procedural.
● The temporal relationship between the matters (are they pending at the same time or close in
time)?
● The significance of the issue to the immediate and long term interests of the clients involved;
and,
● The clients’ reasonable expectations in retaining the lawyer.

The Remedy
If there exists a significant risk of material limitation, then absent informed consent of the affected
parties, the lawyer must refuse the representation of one of the clients or withdraw from one or both
matters.

Danger: When cases are later joined for resolution of inconsistent holdings on an issue of law.
Supreme Court example. (BOTH CASES JOINED, ATTNY HAD TO WITHDRAW FROM BOTH)

Prohibited Transactions
Conflicts can arise not just from relationships with other clients, but also from personal interests of the
attorney. Rule 1.8 seeks to minimize/eliminate this kind of risk by rendering certain kinds of
transactions with clients impermissible or only permissible under limited circumstances. The rule has
11 subsections covering everything from business transactions with a client to sexual relationships with
a client.

Danger of Overreaching
A lawyer’s skill and training together with the relationship of trust and confidence between attorney
and client creates the possibility of overreaching in business, property or financial transactions with a
client.

Business transactions with clients are discouraged and are unethical unless ALL of the requirements of
Rule 1.8(a) are met.
Rule 1.6 Confidential Information
A lawyer shall not reveal info relating to representation of a client UNLESS:
● the client gives informed consent OR
Informed Consent: Written consent after full disclosure. FULL disclosure requires
communication and explanation of the material risks and potential adverse consequences
to the client of any disclosure of confidential information.

● disclosure impliedly authorized by the client OR


Implied Authorization: Implied authorization to disclose exists when disclosure of
confidential information is appropriate in carrying out the representation.

● to prevent reasonably certain death or substantial bodily harm OR


Public policy places emphasis on the value of human life and protection of the public.
When is harm “reasonably certain” to occur so as to permit disclosure under this
exception?
Answer: When it will be suffered imminently; OR there exists a present and substantial
threat that a person will suffer such harm if you fail to take action to eliminate it.

Not an Easy Call: What if the “reasonably certain” standard is met and you “spill the
beans,” but your client does nothing? Answer: You will harm and/or prejudice to the
client because of your misjudgment.

Because of this serious risk, prior to any disclosure, you must conduct a balancing test.
You must carefully balance the public policy considerations behind disclosure v. your
duties of loyalty and confidentiality to your client.

Jailhouse Client Hypo: You are appointed to represent a criminal defendant on a serious
felony offense. You visit/consult with the client at the jail. Client is smug, unconcerned and
nonchalant. You are curious about the client’s cavalier attitude. The Explanation: Client
reveals a detailed plan to “off” the government’s witness. What should you do? What are
your options?

Toxic Waste Hypo: You know that your client has accidentally discharged toxic waste into
the town’s water supply. Can you reveal the information? What is the test? The Test: Does
there exist a present and substantial risk that people who drink the water will contract a life
threatening or debilitating disease? (Rule 1.6(b)(1)); and, Is the disclosure is necessary to
eliminate the risk or reduce the number of victims? (Balancing test).

CAVEAT: Before you make any disclosure under the death/bodily harm exception, first
seek to persuade your client to refrain from the harmful act so as to obviate the need for
disclosure.
Disclosure is last resort!

Immunity From Discipline: A good faith decision to warn can not be re-examined or
second-guessed by disciplinary authorities.

● to prevent the client from committing a crime or fraud that is reasonably certain to result in
substantial injury to the financial interests or property of another (and client has furthered this
by using the lawyer's services) OR after the crime/fraud to mitigate or rectify where the client
used the lawyer in furtherance
Policy Rationale for this Exception: A client who seeks to use your services to commit a
crime or fraud forfeits the protection of attorney - client confidentiality. Keep in mind
that the client can prevent any disclosure simply by refraining from the wrongful
conduct.

Caveat: While disclosure is discretionary, disclosure may be necessary to avoid a


violation of Rule 1.2(d) – assisting your client in the commission of a crime or a fraud.
Remember: Inactive assistance (knowledge with no corrective action) is unethical.

Prior to Enron, this rule didn't exist, only option was “noisy withdrawal.”

● OR to secure legal advice about lawyer's compliance with the Rules OR


a lawyer can get advice about his own personal responsibility in the situation

● to establish claim or defense on behalf of lawyer in a civil claim against client OR defense to
criminal charge OR to respond to allegations in a disciplinary proceeding
includes defending against malpractice and trying to get paid for fees owed

● OR to comply with other law or court order


absent consent from the client, the lawyer should assert all nonfrivolous claims that he
doesn't have to comply with order b/c of the attny-client priv.

GR: in the absence of informed consent lawyer must maintain confidentiality.


Exceptions: You may reveal information relating to the representation of a client to the extent
reasonably believed necessary to:
– to prevent reasonably certain death or substantial bodily harm;

“substantial” means that the harm must be serious, as life-threatening injuries or


illnesses, “and for the consequences of events such as imprisonment for a substantial
period, and child sexual abuse. It also includes a client’s threat of suicide.” Restatement
of the Law Sec 66 at Comment c.

The requirement that the death or bodily harm be “reasonably certain”.


Toxic Waste release example – if the release of the toxic substances into the cities water
supply will likely cause death or debilitating injury to the elderly people in the city then
MAY disclose.

What if your client admits to committing a past crime where another person has been
convicted? Can you disclose? If wrongfully convicted person gets death penalty then
language may allow disclosure to prevent certain death. What if wrongfully convicted
person gets a prison term? If incarceration is “substantially bodily harm”? If it is, then
attorney may disclose, if not then attorney cannot.

You must evaluate the likelihood and sufficiency of injury based upon the facts of the
particular situation!
Example: Person who has no history of violence but is in great emotional turmoil
makes a threat vs someone who has extensive criminal history of violence or
history of psychiatric problems associated with violent behavior

– prevent certain bad acts (in furtherance of a criminal or fraudulent transaction)


It is not necessary for the lawyer to have known that he was part of a criminal or
fraudulent transaction to apply.
Example: client asked lawyer to draft fraudulent loan documents that were
submitted to a bank and the client received the loans.

Example: if a client secures his lawyer’s services in order to file a perjurious


affidavit, the client’s conversations with the lawyer with respect to this affidavit
are not privileged, even if the lawyer did not know at the time the client was
committing perjury.

– get ethics advice;


Example: Attorney has a concern during representation regarding disclosure of client’s
activities so the attorney calls the ethics hotline or another attorney regarding the ethical
duties of the attorney.

– defend an allegation of wrongdoing;


In general, the lawyer may reveal client confidences or secrets if necessary in order to
establish or collect a lawyers fee, or defend herself against a suit or grievance either by
client or by third party based upon the actions of the client.
Purpose is to prevent the client, who is the beneficiary of the fiduciary relationship
(keeping confidences), from exploiting that relationship to the detriment of the
lawyer.

– collect a fee; or,

– comply with other law or a court order.


For example, Model Rule 3.3(b) may require a lawyer to reveal client perjury.
If a court orders an attorney to reveal information the lawyer thinks is protected the
lawyer may reveal the information or challenge the order. A lawyer should assert on
behalf of a client all nonfrivolous claims that other law does not authorize the court’s
order compelling disclosure, or that the information sought is protected (ie attorney-
client privilege) (See MR 1.6 Comment 13). Courts must uphold valid claims of
attorney-client privilege they will not protect other information protected by MR 1.6.

Remember : The lawyer may only reveal facts “to the extent the lawyer reasonably believes is
necessary”! This means considering who to notify – if you know someone who can effectively
intervene, confine your disclosures to that person – and revealing only what is necessary to prevent the
harm.

No Blackmail Allowed You can not threaten to disclose confidential information in order to get a
client to comply with a request!
Example: Client acquitted of assault after confiding to attorney that he was guilty. Attorney
threatened to expose client’s guilt if he did not pay the outstanding attorney fees. No, No, No!!

Post Fraud/Crime You can disclose in situations where you don’t learn of the client’s crime or fraud
until after its commission. In such situations, you have lost the option to prevent the wrong. In
addition, the client has lost the option to refrain from the conduct to prevent disclosure.
Caveat: This exception does not apply if you were not retained until after the fraud or crime
was committed. Why? Not a client at the time the acts were committed. Public has a hard time
understanding this.

Washington State Disciplinary Authorities v Douglas Schafer: Schafer was representing Hamilton in a legal
matter. As an aside, Hamilton confided that he was about to acquire a bowling alley at below market from a
lawyer representing an estate. In exchange, Hamilton was going to buy the lawyer a brand new Cadillac
automobile. Soon thereafter, Hamilton indeed “came into” a bowling alley and, the lawyer for the estate,
Grant Anderson, was “about town” in a shiny, brand new Caddy. At about the very same time, Anderson was
named a Superior Court Judge in Tacoma, Washington. He drove his new Cadillac to the courthouse every
day. Three years later, Schafer had his first case before the Honorable Judge Anderson. Judge Anderson
dismissed the case as frivolous. This made Schafer mad. Schafer reported the three year old bowling
alley/Cadillac transaction to disciplinary authorities. Judge Anderson was removed from the bench. And,
what about Schafer? Holding: Schafer violated the ethical duty of confidentiality to former client. Three
year suspension from the practice of law.

The Duty to Warn


Tarasoff v Regents of the University of California: Psychotherapist knew his patient was planning a
murder and failed to warn the victim. Court held that circumstances called for psychologist to “warn
intended victim or others likely to apprise victim of danger, to notify police, or take whatever steps are
reasonably necessary under the circumstances.” Holding: When a patient presents a serious danger of
violence to another, reasonable care must be used to protect the victim from the intended danger.

The mere suspicion that a client intends to commit a crime or fraud is not enough to permit disclosure.

Belief must have some foundation in fact. (Direct information from the client or some other, reliable
source).

Disclosure Must be Limited: If disclosure is permitted, you may disclose only to the court or to others
that need to know. Appropriate protection orders or other arrangements should be sought to the fullest
extent possible.

Important! The duty of confidentiality survives the termination of the attorney-client relationship, and
the death of the client/former client!

A lawyer must act competently to safeguard info relating to representation of a client against
inadvertent or unauthorized disclose by lawyer or other person subject to lawyer's supervision. When
transmitting a communication, lawyer must take reasonable precautions. Doesn't require special
security measures if the method of communication affords a reasonable expectation of privacy.
Factors: sensitivity of info, extent to which privacy of info protected by law or agreement. A client
can require use of special security measures.

Duty of confidentiality continues after relationship has terminated. Rule 1.9c1 prohibits using info to
the disadvantage of a former client.

Where possible, the lawyer should encourage client to do the right thing so lawyer doesn't have to
disclose.
Harm to be suffered must be imminent unless present threat is substantial.

Lawyer can use hypo to discuss issues relating to representation if no reasonable likelihood the listener
can figure out identity of client.

Lawyers in a firm can disclose to each other UNLESS client has expressly limited lawyers to a select
few.

Confidentiality Hypos and Examples


You are Sorry Sort’s attorney. Sadly, Sorry Sort stepped in front of the 5:10 train three weeks ago in what
police thought was a suicide committed in grief over her step-daughter’s death. The Dilemma: You know
differently. Sorry Sort had confessed to you that she had killed her step-daughter and could not bear the
anxiety of being caught. The state is about to prosecute someone else for the step-daughter’s death.
Ethically, may you tell the state’s attorney what you know?

The answer does not hinge on whether Sorry Sort is dead or alive. Death is not an exception under Rule
1.6(b). No other exception applies thus the information can not be disclosed.

A True Story: 12 year old Erica Baker of Kettering, Ohio, was last seen when she left the house to walk her
dog at about 3 p.m. on February 7, 1999. The dog was later found but Erica was never seen again.
While an Ohio Public Defender was representing one Jan Franks on an unrelated matter, Franks revealed to
the defender that she knew what had happened to Erica Baker. Franks confided that Erica had been struck
and killed by a van and her body buried to cover up the death. Franks further revealed the location of the
body. Franks died in December 2001 without disclosing to authorities anything she knew about Erica Baker's
disappearance.

Ohio Law: Ohio has a special exception to the attorney-client privilege that allows a surviving spouse to
waive the privilege on behalf of a deceased spouse. Franks’ husband waived the privilege so as to permit the
public defender to make the requested disclosures. Lawyer Refuses to Disclose: Although subpoenaed
before a grand jury to disclose any information she may have received from Franks about Erica’s
disappearance, the lawyer refused to disclose anything, arguing that disclosure remained discretionary. The
Court’s Ruling: Lewis must reveal the information from her deceased client. This is an example of situation
where even though disclosure is discretionary, it might be required by other law or court order.

Duty to Safeguard: Safeguard files/confidential records and documents through locking systems.

Train staff on confidentiality. Require an oath of confidentiality (Model Rule 5.3).

Take reasonable precautions transmitting information via fax, unencrypted email, cell phone and voice
mail. Weigh the sensitivity of information.

MR 1.6 vs Attorney-Client Privilege


Attorney-client privilege is an evidentiary privilege – Model Rule 1.6 is an ethical duty
If information is protected by MR 1.6 but not by attorney-client privilege, the lawyer may not volunteer
the information but would have to disclose it in a, subpoena, deposition or other compelled testimony.
If attorney is being forced to testify or provide documents via testimony or subpoena then attorney-
client analysis applies. If court finds that attorney-client privilege does not apply attorney must testify –
cannot refuse based upon MR 1.6 Duty of Confidentiality.
If attorney is gossiping or reveals information outside of a court setting then Duty of Confidentiality
analysis applies.
The Ethical Duty of Confidentiality is Very Broad: It renders confidential ALL information obtained
in the course of the representation regardless of the source.

The Privilege is Very Narrow: It only protects communications between the lawyer and the client; and,
It can only be asserted in court proceedings. Must be asserted by the attorney.

Duty of Confidentiality (COMPARE TO ATTNY-CLIENT PRIV BELOW)


● Attorney’s obligation to protect a client’s confidential and secret information.
● Broader than Attorney-Client privilege.
● Cannot disclose unless client consents except for disclosures that are impliedly authorized to
carry out the representation.
● A lawyer also MAY reveal information to the extent the lawyer reasonably believes necessary:
● to prevent the client from committing a criminal act that the lawyer believes is likely to result in
imminent death or substantial bodily harm
● to establish a claim or defense on behalf of the lawyer in a controversy between the lawyer and
client
● to establish a defense to a criminal charge or civil claim against the lawyer based upon conduct
which the lawyer was involved OR
● to respond to allegations in any proceeding concerning the lawyers representation of the client.
● Not necessary that the source of the information is the client; it is only necessary that the
information is relating to the representation.
Example: A lawyer at a cocktail party, may have learned from client’s loquacious
banker, that the client is delinquent on a major loan and may face bankruptcy. Must the
lawyer keep this information secret (assuming it is not public knowledge)? Yes, the
broad client protection of the rule requires the attorney to keep this information
confidential. However, if this information was “generally known” then usually no duty.

● Applies to former clients through Rule 1.9


● Applies to prospective clients through Rule 1.18.
● Survives client, former client or prospective clients death.
● Applies to any information relating to the representation, whatever its source.
● Attaches to
1. topics covered by the client and attorney in discussing the representation;
2. facts the attorney learns from an adverse party during a deposition;
3. results of scientific studies the attorney has conducted on behalf of the client;
4. scenes depicted in photographs taken by the attorney’s investigator on behalf of a
client;
5. notes of witness interviews;
6. information obtained by outside sources;
7. ANYTHING RELATED TO THE REPRESENTATION!

Attorney-Client Privilege (In brief)


Very narrow.
Applies to communications between the attorney and client (or agents of each) when there is an
expectation of confidentiality.
The underlying information is not protected if it is available from another source, therefore,
information cannot be placed under the evidentiary “cloak” of protection simply because it has been
told to the lawyer.
It is an evidentiary privilege not an ethical duty like MR 1.6
Client can waive if informed consent in writing.

Survives client, former client or prospective clients death.


A client may inadvertently lose the attorney-client privilege. For example:
- If the client voluntarily reveals a portion of a privileged communication with his lawyer,
courts typically find that he may not withhold the remainder.
- If the discussion is held in a non-private place where others overhear.
- Inadvertent disclosure.

Attorney-Client Privilege (in full)


The privilege dates back to the reign of Elizabeth I. It was originally based on the concept that an
attorney should not be required to testify against a client and thereby violate a duty of loyalty owed to
the client.

The privilege exists in every jurisdiction in the United States either by:
- statute
- court rule, or
- common law doctrine.

Policy Considerations Underlying the Privilege:


● The privilege is based on several closely-related policy considerations:
● Clients should be encouraged to be completely truthful with their attorneys so that legal advice
is based on all relevant facts;
● Clients will be reluctant to seek legal advice if they fear that their communications will be
revealed to others; and,
● By encouraging full communication between attorney and client, the privilege promotes
voluntary compliance with laws and regulations.

It is an evidentiary privilege which means that it only arises in the context of litigation. If there is no
litigation in progress, the attorney-client privilege is never applicable. ONLY APPLIES when the
lawyer is being asked to reveal a confidence while testifying; or, the lawyer is asked to surrender a
document or other physical object through force of a subpoena.

Source MUST be the client


To be protected, the information must originate from a confidential communication with the client/
client’s agent, not third persons.

The privilege applies to all written and oral communications between a lawyer and a client; or, between
a lawyer’s agent and the client or the client’s agent. Communications from a non-client/client’s agent,
no matter what vows or other assurances of confidentiality, are outside the privilege. Instead, they are
protected by the ethical duty of confidentiality – Rule 1.6.
Client’s Agent:
Client’s Legal Guardian
Parent of Juvenile

Lawyer’s Agent:
Employees associated with the Lawyer
Investigator
Expert Witnesses
Trial Consultants
PR Agent/Specialists

Requirement for a Confidential Setting:


In order for the privilege to attach, communications from the client must occur in a confidential setting.
Example of a setting that is not confidential is the courthouse hallway.

Inadvertent Waiver: Any public communication of information confided to you, renders the
information no longer protected by the privilege.
Inappropriate revelations to family, friends and acquaintances.
Inappropriate revelations to other lawyers.

Informed Waiver: A client can make an informed consent to waive the privilege. However, such cases
are rare. Get it in writing!

Crime-Fraud Exception: The privilege is forfeited where the client sought or obtained your services to
enable or aid in the planning or commission of a crime or a fraud.
Rationale:
Where the attorney-client relationship is embarked upon in furtherance of criminal activity, and
the relationship is permeated by criminal activity and the client takes an active part in it, the
crime-fraud exception is satisfied notwithstanding that it may have been the attorney who
originally conscripted the client for the illegal purpose.

Washington v Olwell (1964): Facts: On September 7, 1962, Henry Gray and John Warren were in a fight that
resulted in Warren’s being mortally injured by knife wounds. On September 8th, Gray was taken into custody and
admitted stabbing Warren. Gray was not sure of what became of the knife. On September 10th, David Olwell was
retained as Gray’s attorney. After the first conversation with Gray, Olwell came into possession of the knife thought to
be the murder weapon. The coroner then issued a subpoena duces tecum commanding Olwell to appear and produce
the knife. Olwell appeared at the hearing and refused to turn over the knife, claiming attorney-client privilege. The
trial court found him in contempt. Issue: Was the subpoena duces tecum invalid because it required the attorney to
give testimony concerning information he received in confidence from his client? Holding: The subpoena was
invalid; the contempt order was reversed. An attorney should not be a depository for a murder weapon,
but he may retain it for a reasonable time to prepare for the client’s defense. After a reasonable period,
the attorney should, on his own motion, turn the weapon over to the prosecutor. The prosecutor, when attempting to
introduce the weapon at trial, cannot disclose its source. By thus allowing the prosecution to recover the weapon, the
public interest is served, and by prohibiting the prosecution from disclosing the source, the client’s privilege is
preserved. Hence, a balance is reached between conflicting interests. Here the subpoena is invalid because it required
the attorney to testify, without the client’s consent, regarding matters arising out of the attorney-client relationship.
Therefore, the trial court’s order finding Olwell in contempt is reversed and the proceedings dismissed.

People v Meredith (1981): Facts: On April 3, 1976, Wade (the victim) and a friend went to a nightclub. Defendant
Meredith, planning to rob Wade, send defendant Scott into the club to find Wade. When Wade came out, Meredith
attacked him. Two shots were fired, and Meredith ran away. Scott went over to the body; he picked up and hid a bag
containing beer. He later returned for the bag. One month later, after being charged with first degree murder and
robbery, Scott told his attorney Schenk, that he had seen a wallet near Wade’s body. Scott said he picked it up and put
it into the bag with the beer. Later Scott split the money in the wallet with Meredith. After trying to burn the wallet,
Scott threw it into a trash barrel behind his house. Schenk hired an investigator to find the wallet. The PI found it,
Schenk examined it and then turned it over to the police. The prosecutor subpoenaed Schenk and the PI to testify at a
preliminary hearing. At the hearing, Schenk said he received the wallet from the PI but refused to answer further
questions because he learned of the wallet through privileged communications. On a threat of contempt, Schenk said
contact with his client led to disclosure of the wallet’s location. The PI was questioned about finding the wallet. Over
objection, the PI testified that he found the wallet in the trash behind Scott’s house. Issue: Under California law,
does the attorney-client privilege extend to observations that are the product of privileged communications? If so, is
the privilege lost when defense conduct frustrates prosecution discovery? Holding: The attorney-client privilege
extends to observations made as a consequence of protected communications, but if the defense counsel removes or
alters evidence, the location or condition of the evidence is no longer protected by the privilege. Scott’s statements to
Schenk were privileged. Schenks statements to the PI was also privileged. Attorney-client privilege also protects
observations made as a consequence of privileged communications. However, when the defense removes or alters
evidence, it interferes with the prosecutions opportunity to discover that evidence. To extend the attorney-client
privilege to a case in which the defense removed evidence might encourage defense counsel to race the police to seize
critical evidence. Therefore, an exception to the attorney-client privilege is made where defense counsel, or their
agent, has removed or altered evidence. If defense counsel leaves the evidence where it was discovered, his
observations are protected by the privilege. Here because PI removed the wallet, testimony as to its location is
admissible.

Rules from Meredeth


1. If client reveals the location of evidence and the lawyer or lawyer’s agent merely inspects the
evidence without disturbing it, their knowledge of the location of the evidence and observations
are privileged.
2. If the lawyer/lawyer’s agent removes or disturbs the evidence, the original location and
observations are no longer privileged. The privilege has been destroyed.

Meredith represents the balancing of two competing policy considerations: The need for
confidentiality to encourage clients to make disclosures necessary for effective representation vs.
Justice and fairness to the opposing party. (Rule 3.4 prohibits the unlawful obstruction, concealment or
destruction of evidence).

Hypo: What if a new potential client comes in and blurts out “I killed my neighbor!” For whatever
reason, the potential client does not retain your services. What can you do? Model Rules 1.6 and 1.18
indicate that you must keep this information confidential. What happens if you get a subpoena? You
must assert the attorney-client privilege.

Physical Evidence vs. Documents


Objects like weapons and stolen property must be turned over in accordance with Washington v Olwell.

Documents Used in the Perpetration of Crime: Things like written kidnap plans, murder checklists,
ransom notes and maps are treated like physical evidence and must be turned over.

Business Records: Business records used in the commission of a crime are generally treated
differently.
Exception: If the record is the only copy, it should be treated like physical evidence.

Regardless if turned over, business records can not be destroyed and may have to be produced pursuant
to a subpoena.

The Corporate Privilege Several federal cases in the last century have applied the attorney-client
privilege to corporations. This resulted in two competing tests for determining the applicability of the
privilege:
1. the subject matter test; and
2. the control group test.
The Subject Matter Test: Any communication to the corporate lawyer by a person associated with the
corporation is privileged if:
1) The communication is made for the purpose of assisting the lawyer in rendering services/advice
to corporate personnel; AND,
2) The communication relates to the person’s employment.

The Control Group Test: Extended the privilege only to those communications made to a corporate lawyer for the
purposes of rendering services/advice to the corporation where the person making the communication was within the
control group that had managerial responsibility for taking action in the matter relating to the particular legal problem.
Upjohn v United States (1981): For several years there was a split between the two tests. Upjohn case rejected the
control group test as unrealistic and unworkable. Upjohn did use the words “subject matter test” but the test is the
same. Could apply to a janitor if the test is met.

Work Product Doctrine


Protects materials prepared in anticipation of litigation or for trial – your work product. The relation of
the information to litigation is a key, required element.
Hickman v Taylor U.S. Sup Ct (1947) The work product doctrine was first enunciated in this
case. Holding: Unless trial preparation materials are protected, lawyers can not effectively
prepare their client’s case.

Exception: Showing of special need by the party seeking disclosure.

Pursuant to the Doctrine: Your work product is not discoverable unless the court determines that denial
of discovery will unfairly prejudice the party seeking discovery in preparing that party's claim or
defense or will result in an injustice.

Any writing or tangible item that reflects your impressions, conclusions, opinions, or legal research or
theories shall not be discoverable under any circumstances.

Allows lawyers to work and prepare free from intrusion or harassment by opposing parties or their
lawyers.

The only person that can raise work product doctrine as a defense to disclosure of litigation materials is the lawyer.

Rule 1.5 Fees


• Lawyer shall not make an agreement for, charge, or collect an unreasonable fee.

• Lawyer shall communicated the scope of the representation to the client (preferably in writing)
before or within a reasonable time after commencing the representation UNLESS this is a
regular client being billed at the regular rate.
Before any substantive work is performed in the case!

In the case of former clients returning for further services, as long as you are charging
the same rate as before, you are not obligated to re-communicate the rate or basis for the
fee.

The Case for a Writing: The attorney- client relationship is a special kind of contract
involving a fiduciary relationship. You become the client’s agent and have special
responsibilities not imposed on the client. Because of the unique qualities of the
relationship, it is vital to reduce the agreement to writing. You may be able to save
money on malpractice insurance if that is a firm policy!

• Fee may be contingent if allowed. Contingent fees must be in writing and signed by the client.
• Contingent fees aren't allowed in a domestic relations matter for securing a divorce or a
certain amount of alimony or support, or a property settlement. However, it's ok to use
contingent fee in recovery of balances due post-judgment of divorce.
• Contingent fees aren't allowed in criminal case.
• Law may impose limitations on contingent fees such as a ceiling on % allowable or may
require the lawyer to offer an alternative to the contingency fee.

• The agreement must clearly notify the client of any expense for which the client is liable
whether or not he prevails. (telephone charges, copying, etc.)

• Division of fees is ok if 1) in proportion to services performed; 2) the client agrees to the


agreement in writing (including the share amounts); and 3) the total fee is reasonable.
Usually used to bring in a trial specialist

Requirement for Reasonableness


Factors to determine whether a fee is reasonable:
1. time and labor required, novelty and difficulty of the questions involved, skill requisite to
perform the legal service properly
2. likelihood accepting he job will preclude other employment by the lawyer
3. fee customarily charged in the locality for similar legal services
4. amount involved and the results obtained
5. time limitations imposed by the client or circumstances
Hypo: client comes in at 5PM and needs rep in hearing tomorrow morning, you can
charge more $$
6. nature and length of relationship w/client
7. experience, reputation, and ability of lawyer(s) performing the services
8. whether the fee is fixed or contingent

EXAM TIP: if it says on exam that client was charged a reasonable fee, then accept that it is a reasonable fee

G.A. v Thomas Ziegler (1998): Ziegler was the billing partner for Dykema Gossett client Auto-Sense
Limited. Dykema did all of Auto-Sense’s patent work. In 1994, a major investor for Auto-Sense became
concerned over the amount of fees Dykema was charging for the patent work. Auto-Sense hired an
independent patent lawyer to review Dykema’s files and invoices. What the Audit Revealed: Dykema had
charged Auto-Sense $180,000 for services during a two year period. The actual work justified a fee of just
$27,000. The 23 billing statements included charges for services not actually performed and costs not
actually incurred. In the related disciplinary proceedings, Ziegler was suspended for 179 days.

A lawyer can ask for advance payment but is obligated to return any unearned portion. Lawyer may
accept property in payment for services provided interest in the property doesn't accrue while it is the
subject matter of the litigation.

It is proper to define extent of services in light of what the client can pay.

A lawyer shouldn't exploit hourly fee arrangement to rack up charges.


If there is a dispute over fees and a procedure has been established – such as arbitration, mediation,
etc.-- the lawyer must submit to it.

Rule 1.6 Permits you to reveal confidences and secrets necessary to establish or collect a fee. This is
an exception to the duty of confidentiality.

Caveat: You may not charge interest on overdue attorney fees absent client consent! If you have
advance written consent, the maximum interest rate is 7% per annum. If oral consent, the maximum
interest rate is 5% per annum.

Liens
There are two types of lien that a lawyer may utilize to secure a fee:
1. Charging lien.
2. Retaining lien.
Lawyer has right to retain possession of client property coming into lawyer’s hands
during the course of professional employment until the bill is paid. Can hold property as
collateral for payment of fee.

Your primary obligation upon termination of representation is to “take steps to the extent
reasonably practicable to protect [your] client’s interests.” Rule 1.16. Where it may
prejudice the client’s interests, it is a breach of duty to retain a client’s files after
discharge, whether or not you have been paid.

Credit Cards In payment of retainer fees and earned fees.


BEWARE: Commingling! All credit card transactions typically go into the same merchant account.
Reality: Separate merchant accounts for client and lawyer funds is costly and the financial institution
may not allow more than one account. Solution: Limit card payments to earned fees.

Laxton: his firm often has people write two checks, one that goes into client trust fund and the other for general
fund (if fees have been earned)

commingling = strict liablity, you have basically stolen the client's funds

Lawyer will have burden to enforce the fee agreement

The Different Types of Fees


● Hourly fees (Dominant form of fee)
The Requirement for Regular Detailed Billing Statements: Hourly fee clients are
entitled to regular, detailed billing statements. The failure to provide itemized
statements can result in discipline. (G.A. v Ettlinger).
● Fixed or Flat fees
A set amount for an entire case or set amounts for each stage of a case.
Common in criminal cases.
The higher the fixed/flat fee, the greater the risk of scrutiny.
● Minimum fees
Earned on receipt.
A minimum amount that the matter will cost the client.
A minimum fee is billed against an hourly rate.
● Consultation fees.
● Blended/mixed fees.
For example, a one-third contingency fee but if no recovery then an hourly rate or, a 15
% contingency along with a reduced hourly rate.
Must advise the client that other fee arrangements exist.
Like all fees, the overall fee must be reasonable.
● Engagement fees.
Earned on receipt.
A fee paid for the sole purpose of securing your services.
No time is billed against it. It is simply for the privilege of having you as the lawyer.
● Contingent fees.

Contingent Fees
Much debate exists on the question of whether contingent fees are ethical.
While permitted in the U.S., they are not permitted in the United Kingdom and most other common law
jurisdictions.

The Pros
● Provide the poor with access to justice.
● Provide incentive to lawyers to achieve good results for the client.
● Encourage speedy resolution of litigation.

The Cons
● Stir up litigation.
● Potential for rewarding frivolous suits.
● Can result in conflicts between lawyer and client.
● Can result in outlandish attorney fee awards to lawyers.

The ABA’s Position: The benefits of a contingency fee outweigh the risks. It is more important to
ensure that all persons have access to the legal process.

Ethical safeguards…
Rule 1.5(c): An agreement for a contingent fee must be: 1) in writing; AND 2) signed by the client.
The agreement must set forth: 1) the method for determining the fee, including the percentage of
recovery that will go to the lawyer as the fee; AND 2) provisions for the treatment of expenses.

Contingent fees are prohibited in certain kinds of cases like divorce and criminal matters.

When seeking a contingent fee arrangement, you must advise the client of the fact of alternative fee
arrangements.

You may not impose a contingency fee on a client who desires another arrangement.

You may not allow your personal interest in the contingent fee to interfere with your professional
judgment on behalf of the client. (Rules 1.7 and 1.8).

Note: Local court rules may limit the percentage you can recover in certain types of cases.
Example: In Michigan, MCR 8.121 limits a contingency fee in personal injury and wrongful death
cases to one-third of the recovery.
Non-Refundable Fees
Unethical in some jurisdictions, like New York. In re Cooperman, 83 NY2d 465 (1994).

Ethical in Michigan, if specific criteria present and reasonable. MI Informal Ethics Opinion RI-10
The following criteria must be present in order to keep the entire fee when discharged without cause:
1. The case is complex and was likely to preempt other work;
2. The agreement is in writing and is unambiguously clear;
3. The client is of sufficient intelligence; and,
4. The lawyer in fact set aside time, turned down other business and marshaled resources.

Whose Money Is It? If the fee charged is “non-refundable,” to whom does it belong? To the lawyer or
the client?

Fee Collection
Get money up front
Regular billing
fair billing
compromise
law suit against client: the last resort
demand letter example. client is a deadbeat, we're going to withdraw. Enclose a copy of motion to withdraw. This
letter is a last attempt to get paid.

Rule 3.3 Candor Toward the Tribunal


Sometimes, this rule can trump Rule 1.6
● A lawyer shall not knowingly
1. Make a false statement of fact or law or fail to correct a false statement of fact or law
previously made by the lawyer.
2. Fail to disclose legal authority in the controlling jurisdiction known to be directly
adverse to the position of the client and not disclosed by opposing counsel.
TEST: whether overlooked decision is one that should be clearly considered by the court
Policy Rationale: Misleading advocacy is dishonesty toward the tribunal and contrary to
a just and correct result. You can try to distinguish them or challenge their soundness.

3. Offer evidence that the lawyer knows to be false. If the lawyer comes to know of its
falsity, the lawyer shall take reasonable remedial measures, including disclosure.

● A lawyer who represents a client in a proceeding and knows that the person has engaged;is
engaging;intends to engage in criminal or fraudulent conduct related to the proceeding shall
take reasonable remedial measure including disclosure.
Example: A lawyer has a duty to disclose the truth (client’s correct name) to a probation
officer when the client gave a false name (his brothers) to the officer in an effort to
obtain a lighter sentence from the judge and the attorney found out his clients true
identity.

● The duties above continue to the conclusion of the proceeding. (defined as when the final
judgment has been affirmed on appeal or the time for review is past)

● In an ex parte proceeding (such as application for restraining order), lawyer shall inform the
tribunal of all material facts – whether or not they are adverse – so tribunal can make an
informed decision. The object is to yield a substantially just result.
directly adverse = right on-point

Policy Rationale: Misleading advocacy constitutes dishonesty toward the tribunal. In an


ex parte proceeding, one cannot rely on the adversary system to uncover the truth.

This rule applies even to ancillary proceedings like depositions.

The lawyer must not allow the tribunal to be misled by false statements or law or fact that the lawyer
knows to be false.

There are circumstances where failure to make a disclosure is the equivalent of an affirmative misrep.
A lawyer was sanctioned for “intentionally” misleading a California trial court. The lawyer was
not directly asked if his client was dead, and “his answers to the judge’s questions may have
been facially truthful” but that did not save him. This lawyer failed to inform the court of his
client’s death, and represented to the court, during settlement discussion, that he could not
communicate with his client because his client’s “brain was not functioning”. It wasn’t
functioning because his client was dead.

Applies only if lawyer KNOWS evidence is false. A lawyer's reasonable belief that evidence is false
doesn't preclude presenting it to the trier of fact so it can determine whether or not it is false.

FIRM FACTUAL BASIS as to falsity of evidentiary testimony. Other than obvious falshoods, a
lawyer should resolve doubts about the veracity in favor of the client.

The lawyer cannot ignore obvious falsehood.

In some jurisdictions defense attny must allow criminal D to testify in narrative form if attny knows
testimony will be false.

If the lawyer reasonably believes, but doesn't KNOW, testimony is/will be false then attny must allow a
criminal D to testify.

The duties only extend to evidence that you offer. The rule does not require that you correct false
testimony/evidence of the adversary because that is the responsibility of the adversary party.

Also Note: The rule does not prevent you from eliciting false testimony from an adversary party or
witness. There may be tactical reasons exist for doing so such as setting up a witness for impeachment.

Rule 3.3 and Criminal Defendants


Except with regard to the defendant in a criminal case, if you know the evidence is false, you can not
offer it. You must refuse to do so.

Where the source of the false evidence is the defendant in a criminal case, an outright refusal to offer
the testimony violates the defendant’s 6th Amendment rights.

What to do? Easy as 1 – 2 – 3:

1) Counsel not to take the stand. If the defendant insists on testifying, you must accede to the
demand.
2) Counsel to testify truthfully. If the defendant persists in giving false testimony, remonstrate
regarding the consequences of perjury and the remedial measures you will be forced to take.
3) Request permission for a “narrative” so as to avoid assisting the defendant in the perpetration of
a fraud.

The Narrative Approach


People v Johnson: Facts: Defendant Johnson was convicted of numerous violent sexual
offenses, kidnappings and robberies. At an in camera hearing after the prosecution
closed its case in chief, the defense counsel told the trial judge that he had an ethical
problem and could not call the defendant to testify, despite the defendant’s desire to
testify. The trial judge noted for the record that the defendant wanted to testify, but that
the defense counsel would not let him. The defendant did not testify, was convicted, and
received multiple life sentences. Defendant appealed claiming that he was deprived of
his federal constitutional right to testify on his own behalf. Issue: Did the trial court err
in denying the right to testify on his own behalf? Holding: Yes. It should have allowed
the defendant to take the witness stand and tell his story in the narrative fashion.
However, the error was harmless, because the direct and circumstantial evidence against
the defendant was so overwhelming that it could not have been countered by the
defendant’s testimony.

A defendant has a constitution right to testify in a criminal case.

Should try and talk the client out of perjury first.

Some have said that the attorney must try and withdraw. At this point in the proceedings
it would cause a mistrial and would be someone else’s problem.

Refusing a client to testify turns the lawyer into a judge of client’s veracity. Further, it
may require hearing where the lawyer is pitted against his own client. Finally, it
completely denies the client’s constitutional right to testify.

Disclosing the perjury to the court negates the chance the defendant might have changed
his mind and testified truthfully. Again, it may require a hearing where the lawyer is
pitted against his own client.

The narrative approach is not perfect but it is a compromise between the defendant’s
right to testify and the lawyer’s obligation not to present evidence he knows to be false
(suborning perjury). The jury may well believe that some special rule applies to the
defendant and allows his to testify without being questioned. Further the prosecutor can
cross-examine and introduce impeachment evidence and contrary evidence on the merits
– these tools will reduce the chances of successful perjury.
Caveat:

You can only insist on a narrative when you know that your client is going to testify
falsely. A reasonable belief that the testimony is going to be false is insufficient.

Almost every state has adopted some form of the narrative approach
Laxton cannot think of a case where a client has testified in the narrative and been
acquitted.

Reasonable remedial measures:


seek client's cooperation to withdraw or correct the record
if withdrawal from representation is not permitted or will not undo the effect of the false evidence, the
lawyer must disclose to the tribunal as is reasonably necessary to remedy the situation. Even if
disclosure violates Rule 1.6. It is for the tribunal to determine what is to be done then.

Normally a lawyer doesn't need to withdraw unless there is an extreme deterioration of the lawyer-
client relationship due to the falsehood. In that case, it may be necessary to seek permission of tribunal
to withdraw.

What if the Defendant In Fact Commits Perjury


You must take reasonable remedial action!
1. Counsel the defendant to recant.
2. If the defendant refuses, disclose the perjury to the tribunal.

Client is on the stand, starts to lie. Ask for a recess, counsel client to recant. If D refuses disclose the
perjury to the tribunal. Court must then determine: make statement to the jury, order a mistrial, or do
nothing. Even if narrative and client lies you must disclose to tribunal. Probably in a sidebar.

The court then must determine what to be done – making a statement about the matter to the jury, order
a mistrial or do nothing. You must ignore the defendants testimony and not refer to it in other
testimony or in your closing argument. Warning: You can not do nothing! Nor can you withdraw to
avoid the duty to rectify the fraud!

Disclosure is always last resort! Exhaust all lesser alternatives first!

If client lies you cannot withdraw, you must take remedial measures first.

Rule 1.4: Communication


• A lawyer shall explain a matter to the extent reasonably necessary to permit the client to make
informed decisions.
Ordinarily dealing with a comprehending and responsible adult. Where client is a child or
has diminished capacity it may not be possible to fully inform under this standard.

• A lawyer shall:
• promptly inform client of any decision or circumstance where client's informed consent
is required
• consult with client about means of achieving client's objectives
• keep the client informed about the status of the matter
• promptly comply with reasonable requests for information
if prompt response is not possible, inform client when he can expect a response
example: provide an itemized bill
• consult w/client when lawyer knows that the client expects assistance not permitted by
the Rules
Sometimes the exigency of the situation doesn't permit consultation. Lawyer still has to inform client
after he takes a course of action.

When client is an organization, communication is with appropriate officials of the org.

Where many routine matters are involved, a system of occasional reporting may be arrange w/client.

In some circumstances, justifiable to delay transmission of info where client would react imprudently.
Psychiatric diagnoses where psychiatrist says client will react badly.

Rules or court orders may provide info to lawyer that cannot be disclosed to the client.

Rule 1.16: Declining or Terminating Representation


● A lawyer shall not represent a client (or shall withdraw) IF:
● the representation would violate the Rules or other law; OR
differentiate between client demands and client suggestions here
● the lawyer's physical or mental condition materially impairs his ability to represent the
client; OR
● the lawyer is discharged.
A client can discharge a lawyer w/ or w/o cause subject to liability for $ to lawyer
whether a client can discharge appointed counsel is a matter of law
client may not have capacity to discharge lawyer
lawyer should help client consider the consequences

● A lawyer may withdraw IF:


● withdrawal can be accomplished w/o material adverse effect on client's interests
Exception: if client misused lawyer's services in past, he can withdraw
● client persists in action lawyer believes is criminal or fraudulent or repugnant to the
lawyer or lawyer has a fundamental disagreement w/
● the client fails to fulfill an obligation to the lawyer and has been given reasonable
warning that the lawyer will withdraw if not fulfilled
examples: fees, court costs, limited objectives of representation
● representation will result in unreasonable financial burden on lawyer, or has been
rendered unreasonably difficult by the client
● OR other good cause

● the lawyer must comply w/law requiring notice/permission of tribunal when terminating rep.
Tribunal can order lawyer to continue to represent the client.
When in litigation, must ask permission from the tribunal. When lawyer has been
appointed withdrawal requires approval of appointing authority.
G.A. v Leroy Daggs (1971). Holding: Once an attorney accepts a retainer to
represent a client he is obligated to exert his best efforts wholeheartedly to
advance the client’s legitimate interests with fidelity and diligence and, if in
litigation, until relieved of that obligation by the court.

Ruskin v Rogers: Defendant tried to fire his lawyer two days before trial was to
begin and again as attorney was questioning a witness. Judge would not allow a
continuance or substitution of attorneys. Would be prejudicial to other side if
they were ready to proceed.

Court might request explanation and Rule 1.6 may require lawyer to keep explanation
secret. “professional considerations require termination of the representation”

● Upon termination, lawyer will take steps reasonably practicable to protect clients interests such
as giving reasonable notice, allowing time for employment of other counsel, surrendering
papers and property to which the client is entitled, refund advance payment not earned/incurred.
Lawyer may retain papers relating to client to extent permitted by law.
Even if unfairly discharged, must protect client. A lawyer may retain papers as security
for a fee only to the extent permitted by law.

ACCEPTING REPRESENTATION
No Writing Necessary!
Except in the case of a contingency fee, a written fee agreement is not required in order for an attorney-
client relationship to form.

No Fee Need be Paid!


It could be that you take the case pro bono (for free).
The payment of a fee is NOT a prerequisite to the formation of an attorney/client relationship.
All it Takes is an Oral Agreement or Legal Advice!

Warning: For example, should you give advice at a cocktail party and your advice is wrong and the
person relies on that advice to his or her detriment, you have malpractice exposure!

Protect Yourself!
● Non-engagement letters.
● Disengagement letters.
● Closing letters.

Examples of Cases You Shouldn't Take


✗ If taking the case will cause a conflict of interest and will violate a disciplinary rule.

✗ Attorney in such poor physical health she could not do a proper job on the case. In that situation
she would have a duty to reject the case.

✗ Attorney has a substance abuse problem that interfered with his work. He must decline
representation.

✗ Attorney is suffering from depression, that would constitute a “mental condition materially
impair[ing] the lawyer’s ability to represent the client.”

✗ When the legal position is frivolous, meaning, that there is no support in either fact or law.
Also, cannot even make a good faith argument for changing the present law.
- Taste infringement example.

Ending the Attorney/Client Relationship


General Rule #1: An attny may not quit represenation at will, especially if it is a litigation matter (you
are attny of record)
Generally Speaking: Anytime withdrawal would be prejudicial or unfair or would waste judicial
resources or cause undue delay, withdrawal is generally not permitted.

Kriegsman v Kriegsman (1977): Court held that when a law firm accepts a retainer, it impliedly
agrees to prosecute the matter to a conclusion. The firm is not at liberty to abandon the case
without reasonable cause. Plaintiff’s inability to pay the fee is not reasonable cause. Plaintiff
convinced court that she did not have the ability to pay. Further, Plaintiff would have been
seriously harmed if she had to switch lawyers after the Rose firm had put so much time and
effort in the case.

A lawyer may withdraw when:


– there is no client prejudice.
– withdrawal is required under Rule 1.16.
– the client is acting in bad faith.

Holmes v YJA Realty (1987): Defendants hired attorney to handle PI defense. Attorney billed for work
done. At one point, Defendants paid $3,500 that was due, but later an additional sum of $2,275.30
remained due an unpaid over 5 months even though they were able to pay. Attorney asked court to
withdraw because of unpaid bill and client issues. Holmes Court granted motion to withdraw. In
litigated matter, attorney has no absolute right to withdraw. Ethics rules allow attorney to withdraw
when a client deliberately disregards fee agreement and when there is a breakdown of the attorney-
client relationship. Case was not scheduled for trial so ample time to get a new lawyer and no
prejudice. Would be different if it were the day before trial.

General Rule #2: A client can fire his or her lawyer with or without cause at any time.
Policy: Clients must have complete faith and confidence in their legal representation. This is
vital/imperative to the success of our adversarial system.

Exception: Ruskin v Rodgers (1979): Rodgers appealed claiming that trial court erred in
denying his motion to fire his attorney and a continuance of trial. Court held that client does not
have an absolute right to dismiss his attorney at any time with or without cause. In this case,
client tried to dismiss his attorney two days before trial which would have been disruptive and
caused inconvenience to the parties, witnesses and the court. Client did not help himself when
he tried to fire attorney during attorney’s cross examination of witness.

Do You Still Get Paid if You Are Fired Midway Through a Case?
California rule: Can only recover attorney fees where the client succeeded in a recovery.

Rosenberg v Levin (1982) – quantum meruit: fees are limited to the value of actual services rendered
but cannot exceed the amount the attorney would have recovered under the fee contract.

Hypo Apply the Rosenberg case.


A fired attorney is entitled to the reasonable value of the services rendered, but subject to the
top limit – the amount the attorney would have received under the fee contract had the client not
fired him. Reasonable value is $6,000. But if client had not fired him, then he would be
entitled to the larger of $5,000 (the minimum fee stated in the contract) or 20% of the amount
covered by the settlement. Rosenberg says that lawyer cannot come out better by getting fired
than if he stayed on the case. $13,000 settlement minus $1,000 expenses = $12,000
$12,000 x 20% = $2,400. Since $5,000 is larger than $2,400 he gets $5,000 as his fee.

Rule 1.8 Conflict of Interest w/Current Clients: Specific Rules


Rule is to prevent overreaching.

● The prohibitions below are imputed to all members of the firm representing the client (except
Rule about sexual relations).

● A lawyer shall not enter into a business transaction w/client or knowingly acquire a pecuniary
interest adverse to the client UNLESS:
1. The transaction and terms are fair and reasonable to the client; fully disclosed; in writing
in a manner the client can reasonably understand AND
2. The client is advised in writing of desirability of seeking (and given reasonable
opportunity to seek) advice of independent legal counsel on the transaction; AND
3. The client gives informed consent signed in writing to the essential terms of the
transaction.

Prohibits lawyers from buying property from the estate they represent.

Does not apply to standard commercial transactions between lawyer and client where
products are services are ones the client markets to everyone.

When necessary, lawyer should discuss 1) the material risks of the transaction due to the
lawyer's involvement [such as him structuring the transaction or giving legal advice in
such a way that it favors the lawyer at the expense of the client] and 2) reasonably
available alternatives and 3) explain why independent legal advice is desirable.

This rule does not apply to ordinary fee agreements.

The rule does apply when a lawyer accepts an interest in the client’s business or other
non-monetary property as payment of all or part of the fee.

Phillips v Carson: Facts: In 1978, client Phillips retained attorney Carson and his law firm to handle
the estate of her deceased husband. Prior to that time, Phillips and Carson had been personal friends.
Phillips paid the firm $80,000 while the estate was pending to “take care of all her legal business.” In
1980 Carson told Phillips he was having financial problems. She lent him $200,000. Carson gave her
a note and second mortgage on some Arizona property. These documents were executed, and the
mortgage was recorded. In 1981, Phillips lent him $70,000 more. Later, Carson got Phillips to release
the mortgage on the Arizona property and to accept instead a second mortgage on other property.
Carson failed to record the mortgage. Phillips never discussed the loans with any other partner of
Carson’s law firm. In 1982, she found out that the mortgage had not been recorded. She hired a new
lawyer who got the mortgage and recorded it, and she demanded payment of the loans in full. Carson
then filed for bankruptcy under Chapter 11. Phillips sued Carson and his law firm for malpractice.
The trial court granted summary judgment against Carson, individually, but in favor of the law firm.
Issue: Was the trial judge correct in granting summary judgment against Carson but in favor the law
firm? Holding: As to Carson, the judge was correct. As to the law firm the judge was incorrect.
Reasoning: There were no disputed facts concerning Carson’s own liability. He breached his duties
as an attorney by failing to record the mortgage, failing to properly advise Phillips, and failing to
recommend that she seek outside counsel before lending him money. There were unresolved issues
of fact regarding the firm’s liability. Did Carson have apparent authority to bind the firm? Were the
loans from Phillips to Carson apparently authorized by the firm? Were his acts and omissions within
the usual course of the firm’s business? Were they actually authorized by the firm? In light of these
unresolved fact issues, it was error for the trial judge to grant summary judgment in favor of the firm.

G.A. v Hosler (1995) - Approached client for $25,000 loan. Obtained loan from client with interest
rate at 7% - a rate that was lower than the fair market rate and failed to inform client that she could
include and enforce a demand clause requiring immediate payment upon default; failed to give client
reasonable opportunity to seek advice from independent counsel regarding the loan; and failed to get
client’s written consent to the loan.

G.A. v Johnson (1994) - Asked a long-time client to loan him $100,000 to be repaid in a one to two
year period. The client loaned him the money, and he prepared and executed a Real Estate Sale and
Repurchase Agreement, warranty deeds and an Assignment of Land Contract, to secure the loan. He
failed to advise his client to seek the advice of independent counsel and/or a financial advisor
concerning the loan; failed to record the documents he prepared as security for the loan; failed to
advise his client to record the documents in question; and failed to timely repay the loan.

● A lawyer shall not use info relating to rep. To the disadvantage of the client unless he's stupid
enough to give informed consent.
Doesn't apply to case where lawyer uses research from one client's case to benefit
another.

This rule does not prohibit uses of info that do not disadvantage the client.

It is important to note that the lawyer violates this rule even if the lawyer does not
disclose any information to anyone. What is relevant is that the lawyer uses the
information to the client’s detriment.

Example #1 assume that Lawyer A leaned, in confidence, that Client is planning to buy
certain farm land in order to build a new shopping center. Lawyer then secretly visits the
farmer and buys the land, but does not reveal to the farmer any of the Client’s secret
information. Lawyer plans to raise the price in reselling the land to Client, because he
has learned, in confidence from the client, that his location is very important to the client
– far more than the farmer suspects. See comment 5 MR 1.8 – lawyer may not (or have
third party) purchase one of the parcels of land that client intends to purchase.

Example #2 Assume that Lawyer A leaned, in confidence, that Client is planning to buy
certain farm land in order to build a new shopping center. If Lawyer A were to purchase
a parcel adjoining or adjacent to the land, that may or may not be a violation. It would
depend on the clients expectations - whether the client had indicated an interest in
purchasing that land as well (or the plans would call for potential expansion) – and
whether the purchase would disadvantage the client in some respect. I would disclose to
the client and obtain his consent prior to purchase to prevent client later claiming it was
his intent to purchase the adjacent or adjoining property.

● A lawyer shall not solicit a substantial gift from the client (including testamentary gift) or
prepare an instrument on behalf of the client to give the lawyer (or someone related to him) a
substantial gift UNLESS the lawyer/recipient is related to the client. Related = spouse, child,
grandchild, grandparent, parent, or other relative/individual w/whom the client maintains a
close familial relationship.
Lawyer can accept a gift if it meets general standards of fairness like a holiday gift or
token of appreciation. Substantial gifts may be voidable by client under doctrine of
unfair influence which treats client gifts as presumptively fraudulent.

Rule doesn't prohibit a lawyer from seeking to have lawyer or partner or associate
named as executor or to another potentially lucrative fiduciary position. Lawyer should,
however, advise client concerning the nature and extent of lawyer's financial interest and
advisability of alternative candidates.

Policy Concerns: Undue influence. Overreaching. Imposition on clients.

If the client offers a more substantial gift, you may accept it although it might be your
policy not to do so.

If the client does want to make a substantial gift and the delivery requires the drafting of
a legal instrument, then the lawyer may not draft that instrument. The client should
obtain neutral and detached advice from another lawyer. The client may not use another
lawyer in the disqualified lawyer’s firm.

If the gift is substantial but does not require an instrument, and if the lawyer does not
solicit it, then the text of MR 1.8(c) is inapplicable. However, the comment adopts a
“general standards of fairness” test – See Comment 2.

The burden is on the lawyer to demonstrate the fairness of the gift. The failure to meet
this burden would likely lead a court to find the gift voidable by the client on the
grounds of the doctrine of undue influence.

If the lawyer cannot accept or solicit a substantial gift for herself, then the lawyer may
not solicit the gifts for persons related to her including “spouse, child, grandchild,
parent, grandparents or other relative or individual (ie. fiancée, life partner, boyfriend,
girlfriend) with whom the lawyer or the client maintains a close, familial relationship.”
This prohibition is necessary to prevent lawyers from getting around the rule by
structuring client gifts to related persons: some people launder money; others launder
bequests. There are two exceptions to this rule. One is where the lawyer and the client
are related under the definition of relationship above. The second is where the lawyer is
seeking to be appointed as executor of the client’s assets or similar lucrative fiduciary
position. Such conflicts are handled under the general conflict of interests (MR 1.7).

● A lawyer shall not negotiate an agreement where the client gives the lawyer literary or media
rights to portray the representation. [No made for tv movie rights.]

Query: What if the media has approached you, during your representation of a client,
with potential book and movie deals about your life? The client, while not the primary
focus of the media interest, would also receive compensation for his “life rights.” Does
1.8(d) apply? Answer: Rule 1.8(d) does not apply. That rule prohibits a lawyer from
acquiring literary/media rights based substantially on a client’s matter prior to
conclusion of the case. The rule does not prohibit a lawyer, even during the pendency of
a client’s case, from making an agreement with respect to his own media rights.
Nevertheless, there remains a potential conflict of interest under Rule 1.7(a)(2) – the
deal could interfere with your independent professional judgment on behalf of the client.
Therefore, client consent must be obtained optimally after independent legal advice.

There is a danger that the lawyer, in bargaining with the client, may overreach. For example, needing
the lawyer’s services, may feel pressured to waive the attorney-client privilege so that the attorney
can later write a more interesting book. This rule prohibits that problem. Also, the lawyer, in his
representation of the client, may make decisions that will enhance the value of the publication rights
to the detriment of the client – pressuring client to rejecting the plea bargain and go to trial rather
than properly advising client to accepting a plea bargain when there is little chance at trial. This rule
protects not just the client but the interests of the judicial system in having competent and
independent representation.

Once the relationship has ended, the lawyer has not special powers or advantages in negotiating any
publication rights from his former client. The lawyer would be in no better bargaining position than
any other person who wanted to write a book on the client’s legal adventures.

● A lawyer shall not provide financial assistance to client in connection w/pending/contemplated


litigation EXCEPT:
1. lawyer may advance court costs and expenses of litigation, repayment can be contingent
on outcome.
2. Lawyer representing indigent may pay court costs and expenses of litigation on behalf of
the client.

Lawyers cannot subsidize lawsuits through loans b/c would encourage suits that might
otherwise not be brought.
Can pay for medical examination and costs of obtaining and resenting evidence.

G.A. v Funk (1999) - $175 to pay for prescription medication – 180 day
suspension.

Cleveland Bar Assn v Nussbaum (2001) - $26,000 to a client/friend for surgeries


needed after motorcycle accident – public reprimand after a 29 year unblemished
record.

A lawyer may not advance or pay a client’s living or general medical expenses because these
expenses are not permitted within the permitted court costs and expenses of litigation category.

In some cases, the client’s injuries prevent her from earning sufficient income to support her family.
Consider, for example the situation where the client is threatened with eviction from an apartment,
and she lacks the funds necessary to pay basic sustenance expenses for her family. Assuming,
moreover, that the client has an excellent chance of prevailing in the personal injury action because
of the defendant’s liability. The lawyer wants to provide a loan, buy groceries for the family, or pay
for medicines that the client needs. This type of financial assistance is prohibited under the Model
Rules. The drafters made it clear that the rules prohibit “making or guaranteeing loans for living
expenses, because to do so would encourage clients to pursue lawsuits that might not otherwise be
brought and because such assistance gives lawyers too great a financial stake in the litigation.” See
Comment 10. This has been interpreted by some courts to include financial assistance no matter how
small.

There has been a historical aversion to lawyers assisting clients financially based on what the
common law called “maintenance,” “champerty” or “barratry.”
Note: This rule has been interpreted in many, many different ways throughout the country – some
have applied sections broadly while others have applied the same sections narrowly.

There are a couple of exceptions to this rule. Expenses of costs of litigation (may include court costs,
deposition fees, copying costs, transcripts, necessary medical examination to determine the extent of
client’s injury, expert witness costs, costs to transport client to and from trial, etc.) may be advanced
and the lawyer can agree that the client’s obligation to repay these amounts is contingent on the
outcome of the matter. Second, if the lawyer represents an indigent, the lawyer may pay court costs
and expenses of the litigation on the behalf of the client. This rule helps indigents who have suits
where they are unlikely to collect money – ie. the relief sought does not result in a monetary
payment.

Exception: Living Cost Loans


You may be able to assist a client in obtaining a cash advance from a litigation funding
organization (the funding group advances the $ in exchange for an interest in the client’s case
and is repaid from the recovery). Okay if:
1) You have no personal interest in the agency;
2) You guard against predatory lending practices (deal is fair);
3) Attorney-Client confidentiality is protected; and,
4) There is no interference with your independent professional judgment.

● A lawyer shall not accept compensation from someone other than the client UNLESS:
1. the client gives informed consent AND
2. there is no interference w/lawyer's independent professional judgment or the attny-client
relationship AND
3. confidentiality is maintained under 1.6

● A lawyer who represents two or more clients must get their informed consent in a signed
writing before making an aggregate settlement or in a criminal case an aggregated agreement as
to guilty or nolo contendere pleas.

In class action suit, lawyer must comply with rules regulating notification of class members
and procedural requirements. However, in most class action lawsuits the judge controls the
communication to the class.

The purpose of this rule is to require that all lawyers, representing multiple clients in an
aggregate settlement provide each client with sufficient information to allow the client to
decide whether or not to accept the settlement. Such basic information must include the
settlement offered to the other clients. If the opposing party makes a settlement offer to
only one client at a time, the aggregate settlement rule does not apply. However, such
information may need to be shared to the other clients under MR 1.7 because the
lawyer’s failure to do so materially limits the lawyer’s representation of the other client
who does not have the information about the settlement offers made to the other clients
of the lawyer.

The rule does not specify what types of information must be given by the lawyer to the
individual client to satisfy disclosure to multiple clients. But a lawyer, at a minimum
must disclose the total amount of the aggregate settlement or the result of the aggregated
agreement; the existence and nature of all claims, defenses and pleas involved in the
settlement or agreement; the details of every other clients participation in the settlement
or agreement; the total fees and costs that will be paid to the lawyer as a result of the
settlement or agreement if the fees and/or costs will be paid in whole or in part from the
proceeds of the settlement or agreement; the method to which costs are to be
apportioned amongst the clients.

● A lawyer shall not:


● make an agreement prospectively limiting lawyer's liability for malpractice unless client
has another lawyer when he makes the agreement OR
● settle a claim or potential claim for malpractice liability unless the client/former client is
advised in writing of the desirability to seek, and is given a reasonable opportunity to
seek, independent legal counsel.

EXAM TIP: Look for independent representation

A lawyer can still form a limited liability entity.

An agreement to settle still ok.

Rule 1.8(h) does not prohibit you from entering into an agreement with a client to
arbitrate malpractice claims provided:
1) the agreement is enforceable; and
2) the client is fully informed regarding the scope and effect of the agreement.

Note: The attorney may not condition a fee settlement on a client’s agreement not to file a
disciplinary charge. That would violate the lawyer’s fiduciary obligation.

Policy Behind This Rule: Differences in willingness to make or accept an offer of settlement
are among the risks of common representation of multiple clients. Before any settlement offer
or plea bargain is made or accepted, you must inform each client about all the material terms
including what the other client will receive or pay.

The rationale behind this principle is that there is a risk that the first lawyer will overreach
unless a second (non-conflicted) lawyer independently represents the client in making the
decision. Advising the client to seek assistance of another lawyer is not enough. If the client is
not actually represented by another lawyer, then there cannot be a prospective waiver

There are very few scenarios where the second lawyer would advise the client to waive his right
to malpractice against the first lawyer prospectively. It could open the door to malpractice on
the part of the second lawyer.

Example: in house counsel wants independent opinion about a tax matter, outside
counsel wants a waiver b/c otherwise he's got to do his due diligence and look through
corp's records. In house doesn't want that expense so they waive.

● A lawyer shall not acquire a proprietary interest in the cause of actions/subject matter of
litigation EXCEPT:
● lawyer can acquire a lien to secure his fee/expenses
● lawyer can contract for reasonable contingent fee in civil case
If lawyer had an interest, he would be harder for client to discharge. Also, in some cases
lawyers will want to pursue their interests far beyond where the client would stop. There
is a risk that the lawyer will not exercise independent professional judgment for his
client.

There are two basic types of liens- retaining liens and charging liens. We have discussed
these two types of liens in the past.

The other exception regards contingency fee agreements in civil case. However, the
lawyer’s interest in the case cannot be that of a co-plaintiff. In other words, the client
may not assign to the attorney part of his cause of action in way that would allow the
lawyer to prevent settlement. The client cannot waive his right to decide when to settle
litigation.

For example, assume that Client agrees to compensate Attorney A by giving her a ¼
interest in certain real property and mining claims. Ownership of these properties is
disputed and A defends Client (and A herself as well, to the extent that A’s on ¼ interest
is involved). Client becomes dissatisfied with A’s services and tries to discharge A, who
refuses to leave because of her interest in the litigation. A has violated the ethics rules by
refusing to accept the client’s discharge.

● A lawyer shall not have sexual relations with a client unless a consensual relationship existed
before the attny-client relationship commenced.
Fiduciary relationship almost always unequal. Emotional entanglement makes it harder
to represent. Also blurs line of when attny-client priv applies b/c confidences are
protected ONLY when imparted in the context of the attny-client relationship. Lawyer
can have a non-sexual relationship but must consider whether his ability to represent will
be materially limited. When client is an organization, the lawyer cannot have sex
w/employee who supervises, directs, or regularly consults w/lawyer about the
organization's legal matters.

Further, the lawyer open himself or herself up to a potential malpractice claim. For
example, a Rhode Island jury awarded a client $25,000 compensatory and $250,000
punitive damages for malpractice because of her sexual relationship with her lawyer.
She claimed that the lawyer handling her divorce case threatened to work actively to
lose custody of her child if she did not engage in sex with him. The court did not require
that the plaintiff prove that any adverse legal work was performed. In fact, she was
awarded custody, alimony, child support and 60% of the marital assets.

Rule 1.9: Duties to Former Clients


● lawyer shall not represent another person in same or substantially related matter in which that
person's interests are materially adverse to the interests of the former client UNLESS the
former client gives informed consent confirmed in writing.
● If a lawyer's former firm represented someone, a lawyer shall not KNOWINGLY represent a
new person in the s.or.s.r.m.i.w.t.p.i.a.m.a.t.t.i.o.t.f.c. AND about whom the lawyer had acquired
info protected by 1.6 this Rule that is material to the matter UNLESS the former client gives
informed consent confirmed in writing.
Addresses the situation of lawyers who, while working for a firm, gain confidential
information about a client’s matter. If a lawyer leaves the firm, the lawyer can’t
represent someone against that client “in the same or a substantially related matter”
absent the former client’s informed consent, confirmed in writing. NOTE: The rule
applies only where actual knowledge of protected information is demonstrated.

In Other Words:
The lawyer who leaves but has NO confidential information about a matter, will not
cause the disqualification of his/her new firm from representing the other side in the
matter. “Empty head = full pockets & Full head = empty pockets”

● A lawyer/lawyer's former firm who represented a client shall not thereafter use info relating to
the rep to the disadvantage of the former client (except as rules require or when info is
generally known) or reveal info relating to the former rep.

Can't draft a K for one client and then turn around and try to have the K rescinded for another.

All affected clients must give their informed consent.

Lawyer's involvement w/former client is a question of degree.

The “matter” depends on the facts.

Matters are “substantially related” if they involve the same transaction or legal dispute or confidential
info from former rep. Would materially advance new client's position in subsequent matter.
Info that has been rendered obsolete by passage of time may help determine whether reps are
substantially related.
Example: Cannot represent a guy regarding his finances and turn around and help his wife with
their divorce.

Info disclosed to the public isn't protected.

This rule operates to disqualify the lawyer only when the lawyer involved has actual knowledge of info
protected by Rules 1.6 and this Rule. If imputation were applied rigorously then it would curtain
lawyers' freedom to move around.

Distinguish between lawyers with general access to all files in a firm and those who only have access to
files of a limited number of clients. Burden of proof should rest upon firm whose disqualification is
sought.

Rule 1.9(b) prevents a departing lawyer with confidential information from taking cases against clients
of the lawyer’s former firm.

The passage of time may render any information gained in a prior representation obsolete.

Rule 1.9 governs our continuing duties to clients after the representation has ended.

Conflicts with former clients can ALWAYS be cured with consent.


Step by Step Rule 1.9(a) Analysis:
Step One: Is the matter against (materially adverse to) a former client? If yes, go to Step Two. If no,
there is no conflict.

Step Two: Is it the same matter as the former client’s matter or a substantially related matter? If yes,
go to Step Three. If no, there is no conflict. Substantially Related Matter: Whether the new matter is
“substantially related” to the former matter is a question of fact and degree. Apply the “Substantial
Relations” test.

Substantial Relations Test:


Is there a reasonable probability that confidences were disclosed which could be used against
the former client? (Are there facts gained in the prior matter that could be used against the
former client in the current matter)? If so, the matters are similar enough that a substantial
relationship is presumed.

The test is met if the facts of the two matters are similar whether or not the lawyer received
confidential information from the former client.

The Substantial Relations presumption is rebuttable.


Example: Where the lawyer can demonstrate no personal involvement in the prior matter
and that he/she did not receive any confidential information disqualification will not be
required.

Mere general knowledge of a client’s policies and practices is not enough to disqualify the
lawyer.

There must be a demonstration of specific facts gained in the prior relationship that are relevant
to the current matter.

Haagen-Dazs v Perche, etal (1986): Facts : Double Rainbow and Perche No! make super premium
ice cream. They sued Haagen Dazs for violating anti-trust laws. Pillsbury is the parent corporation of
HD. Pillsbury and HD moved to disqualify DR’s Minn counsel (“Gray”) and its SF counsel
(“Alioto”) on the ground that Gray had employed a lawyer named Jesse who had formerly been on
Pillsbury’s in-house legal staff. They claimed that while Jesse was a Pillsbury lawyer he had access
to confidential information about HD distribution policies and other matters relevant to the anti-trust
claims. Jesse worked mostly on international business matters. These included (a) a joint-venture
through which HD sought to introduce its ice cream in Japan; (b) an Australian trademark
registration for HD; (c) and a HD sales arrangement in Singapore. Further Jesse was still in the
Pillsbury legal department when the department was defending an anti-trust claim brought by Ben &
Jerry’s involving issues similar or identical to those in the present litigation. Pillsbury argues that if
Jesse himself is disqualified, then the entire Gray firm must be disqualified also. Further, Pillsbury
argues that the Gray firm and the Alioto firm have a long-standing relationship as associated counsel
in this and other anti-trust cases and that the two firms regularly exchange information, strategy and
advice. Thus, Pillsbury argues the Alioto firm should be disqualified as well. Issues: Must the Gray
firm be disqualified if one of its attorneys previously worked on substantially related matters while in
the employ of the adversary? Assuming Gray firm is disqualified, must the Alioto firm also be
disqualified because of its close relationship with the Gray firm? Holding: The Gray firm must be
disqualified because of Jesse’s former work for Pillsbury, the present adversary. The Alioto firm
need not be disqualified because the disqualification of the Gray firm will almost entirely eliminate
the possibility of disclosure of Pillsbury’s confidences. But the Alioto firm must return to the Gray
firm whatever documents and other information it already received from the Gray firm.
Reasoning: Analysis: MR 1.9(a) prohibit a lawyer from representing a client who is adverse to a
former client if the lawyer earlier represented the former client in the same or substantially related
matter (unless the former client consults after informed consent).
The substantial relationship test is met if the facts of the two matters are similar, whether or not the
lawyer received confidential information form the former client.
If there is a reasonable probability that confidences were disclosed which could be used against the
client in a later, adverse representation, a substantial relationship is presumed.
This is a rebuttable presumption – showing that he had no personal involvement in the substantially
related matters and that he or she did not in fact receive confidential information.
Here, Jesse did work on substantially related matters. He was with Pillsbury for 10 years. He had
intimate knowledge about HD. Further, he was in the legal department when the department was
handling the defense of the Ben and Jerry’s case.

Step Three: The only way to proceed with the representation is to obtain the former client’s informed
consent, confirmed in writing.

MODEL RULE 1.9 REVIEW

Model Rule 1.9 governs conflicts that arise out of the fact that a particular lawyer previously
represented one client and is now representing a different client in a matter adverse to his former client.

Rule 1.9 governs the situation where the lawyer is personally disqualified, i.e. individually disqualified.

Rule 1.10 focuses on when the law imputes an individual’s conflict of interest to other lawyers in that
individual’s firm.

The general tenet of Rule 1.9 is that unless there is a proper waiver by the client, the lawyer should be
disqualified from taking a matter adverse to his former client if the lawyer acquired confidential client
information from the former client that is material to the new representation.

Rule 1.9 disqualifies the lawyer if he now represents a person in a matter that is “the same” or
“substantially related” to the past representation of another client and the new client’s interests are
materially adverse to the interests of the former client.

While Rule 1.9 deals with the individual lawyer’s disqualifications, Rule 1.10 deals with imputed
disqualification. If one lawyer in a law firm is disqualified, others associated with the affected lawyer
may be automatically disqualified. Some disqualifications are imputed and others are not.

Rule 1.10 lays out the basic rule regarding imputation.

Rule 1.10 always imputes to all members of a law firm all of the disqualifications of Rule 1.9. Thus,
lawyer A may be disqualified because he has actual confidential knowledge about the former client that
is material to the present situation. As long as lawyer A is within a given law firm, that law firm is also
disqualified because of Rule 1.10(a), which governs imputation of conflicts. If lawyer A is the only
person with confidential knowledge leaves that law firm, lawyer A is still disqualified, but A’s former
law firm is no longer disqualified since the only reason the original law firm was disqualified is
because of the knowledge of lawyer A who is no longer there.
A. Model Rule 1.9(a)

Rule 1.9(a) is fairly straight forward. If the lawyer used to represent a client in a particular matter, but
no longer does so, that lawyer is still disqualified from representing a new client in that same matter if
that new client has interests “materially adverse” to the former client. The lawyer, in short, may not
switch sides. The lawyer also may not take a matter adverse to the former client if the new matter is
“substantially related” to the former matter. Only written informed consent can allow the subsequent
representation to take place.

“Substantially related” means if they involve the same transaction or legal dispute, or if there is
otherwise a substantial risk that confidential factual information as would normally have been obtained
in the prior representation would materially advance the client’s position in the subsequent matter. See
Rule 1.9 Comment 3.

Example: A lawyer has represented several auto dealerships over the years in judgment/collection
matters. However, a year ago, she resigned from representing any auto dealerships, although she
practices in the same locality. A debtor now asks her to represent him in a collection matter that one of
her former auto dealerships is bringing. Lawyer has stopped representing any of the auto dealerships.
Assume the new collection suit is a matter totally unrelated to her previous clients, i.e. no confidential
information learned in the previous representations would be relevant to this case. In this circumstance,
her former clients may not prevent the new representation. Lawyer does not have to secure the consent
of her former clients. Further, information gained through representation is sometimes said to have a
limited shelf-life. That is the length of time between the end of the lawyers former and current
representations may render information obtained in the former representation obsolete and no longer
materially relevant. However, mere length of time is not dispositive if information is still relevant.

B. Model Rule 1.9(b)

Rule 1.9 (b) provides that if the lawyer was with a former firm and that firm had previously represented
a client in the matter, the lawyer may not ethically represent a new client with interests adverse to the
client of the former firm if the lawyer had acquired secret information about his former client, i.e.
information protected by Rules 1.6 or 1.9(c). The former client can waive his or her rights, but only
after informed consent in writing.

Example: Assume that lawyer A is a member of firm #1. Firm #1 represents client P in a suit against
client D. Attorney A herself was not involved in any representation of client P and A has not acquired
any secret or confidential information from client P or from firm #1 members that is relevant to client
D. A now leaves firm #1 and joins firm #2, which represents client D. Note that P and D’s interests are
materially adverse to each other, and firm #2 (A’s new firm) is representing D in a matter that is the
same as (or substantially related to) the matter in which firm #1, A’s old firm, is representing P. In this
case, the substantial relationship test is met. However, both firm #2 and A may properly represent D
against P because A acquired from client P no material information protected by Rules 1.6 or 1.9(b)
thereby successfully rebutting the presumption of disqualification. For A an empty head means full
pockets.

Now assume the same facts in the previous example except that A personally worked for P in the case
of P v D while A was with firm #1. Let’s assume that A did not acquire any material client secrets or
confidences, but that she did work on the particular case. For example, she filed an Appearance in court
and asked for a continuance. Unless the former client consents, Rule 1.9(a) personally disqualifies A.
Rule 1.9(a) provides that if A formerly represented a client in a matter, she may not thereafter represent
another person in the same matter or a substantially related matter if the other person’s interest is
materially adverse to the interest of the former client, unless of course, the former client consents.

The next question is whether A’s disqualification is imputed to the other lawyers in firm #2. Rule 1.10
is the general rule in imputation and subsection (a) in principle imputes conflicts under all of Rule 1.9.
Thus it appears that Rule 1.9(a) disqualifies firm #2, unless the firm obtains a written consent from the
former client.

C. Using client’s information to the client’s disadvantage

The Rules forbid using client secrets or confidences to the disadvantage of the client. This prohibition
applies equally to protect a former client unless the information has become generally known. Rule
1.9(c)(1).

A lawyer violates Rule 1.9(c)(1) even if the lawyer does not disclose the information to anyone. What
is relevant is that he uses the information to the client’s detriment. We have discussed this issue in Rule
1.8.

For example, suppose that the lawyer learned in confidence that the client is planning to renew the
lease on the building that the client now uses. Lawyer then secretly visits lessor and purchases the lease
for the lawyers own account, but does not tell lessor any client information. Lawyer plans to raise the
rent because he has learned in confidence that the location is more important to client than the lessor
suspects. Lawyer has committed a violation.

Rule 1.9(c)(2) separately prohibits a lawyer from revealing a former clients secret unless other rules
allow or require revealing the information.

Waiver-Rules 1.9(a) and (b) allow a former client to waive the conflicts of interest. If there is a
conflict, an otherwise disqualified lawyer may represent a new client against the former client if the
former client gives informed consent in writing.

Rule 1.9(c) allows for waivers as well as confirmed by comment 9.

D. Prospective waivers

If a former client may waive a conflict, he should also be waive his objections prospectively. If the
prospective waiver meets all the requirements required when the former client grant’s consent to a
present conflict of interest. The lawyer that relies on this prospective waiver must show that it has
“reasonably contemplated” the future conflict so that the client’s consent is reasonably viewed as fully
informed when it was given.

In other words, a lawyer must give the client or former client the information in a way that this client or
former client will appreciate the significance of the waiver that is being sought. Otherwise, it is not
informed. It is unlikely that the court would uphold a waiver that purports to allow disclosure or use of
client confidences against the client unless the client very explicitly agreed to such disclosure or use. In
other words, it is much easier for a client to waive prospectively a conflict based on loyalty, but it is
much, much harder to waiver prospectively a conflict based on breaches of confidences because a
client cannot normally know at the time of the prospective waiver what those confidences might be or
what there relevance and significance will be at some point in the future.

Rule 1.2: Scope of Representation and Allocation of Auth. between Lawyer and Client
This rule confers the ultimate auth. Upon the client. Typically client defers on technical legal
and tactical matters while lawyers defer to client on expense to be incurred and concern for third
parties.

If a lawyer comes to KNOW or reasonably should know client expects assistance not permitted
by the Rules or other law, OR lawyer going to act contrary to client's orders, lawyer must
consult w/client.

● A lawyer shall abide by client's decisions concerning objectives of representation.


● Shall consult w/client as to means of pursuing objectives.
● May take action as is impliedly authorized.
Absent a material change in circumstances. Sometimes a lawyer gets advanced auth, but
this auth can be revoked.
● Shall abide by client's decision whether to settle a matter.
● In criminal case, shall abide by client's decision as to pleas, whether to waive jury trial, whether
client will testify. MUST abide by a client’s decisions in a criminal case!
● A lawyer's representation of client does not constitute an endorsement of the client's political,
economic, social, or moral views or activities.
● A lawyer may limit the scope of the representation if the limitation is reasonable under the
circumstances and the client gives informed consent.
Agreement for limited rep does not exempt lawyer from duty to provide competent rep.
● A lawyer shall not counsel a client to engage or assist client in conduct that is criminal or
fraudulent.
Active Assistance
Saunders Dorsey + Drug Money Laundering = Disbarred!!
Inactive Assistance
G.A. v Dennis Koltunchuk – Subornation of perjury in a deposition before the
U.S. Attorney.

● A lawyer may discuss the legal consequences of ANY proposed course of conduct w/client.
Critical distinction between presenting an analysis and recommending means by which a
crime/fraud may be committed w/impunity.

A lawyer may not continue assisting a client in conduct originally supposed to be proper but
then discovered to be criminal or fraudulent.
In some cases withdrawal alone not enough. May require notice of the fact of
withdrawal and to disaffirm an opinion, document, affirmation or the like.

● May counsel or assist client to make good faith effort to determine the validity, scope, meaning,
or application of law.
To determine the validity of a statute or regulation might require a course involving
disobedience of the statute/reg or of the interpretation of it used by govt authorities.
Rule 1.15: Safekeeping Property
● A lawyer shall hold property of clients or third persons separate from lawyer's own property.
Funds shall be kept in a separate account maintained in same state as the lawyer's office
(elsewhere w/consent). Other property shall be identified and safeguarded. Complete records
shall be kept and retained for 5 years after the termination of rep.
● A lawyer may deposit his own funds in client trust account only to pay for bank service charges.
● Legal fees and expenses paid in advance must be put in client trust account and can only be
withdrawn by the lawyer as the fees are earned/expenses incurred.
● Upon receiving funds or other property, lawyer shall promptly notify the client/third person to
whom it belongs. Without an agreement to the contrary, the lawyer must propmpty deliver the
property. The client/third person is entitled to request and receive a full accounting of property
and lawyer must promptly comply.
● When two or more persons claim ownership of the same property (even lawyer in fee dispute)
the property shall be kept separate until the dispute is resolved. The lawyer shall promptly
distrbiute all property where interest in property is not in dispute.

A lawyer shouldn't unilaterally assume to arbitrate a dispute.

A lawyer should participate in a lawyer's fund for client protection (to reimburse clients who have been
cheated by lawyers) even if not required to participate.

IOLTA Accounts (Interest on Lawyers Trust Accounts)


Allows Lawyers to deposit nominal and short-term client or third party funds into a pooled interest or
dividend bearing trust accounts. If funds are long term you should put them into a separate account for
the benefit of the client. The net interest or dividends generated on IOLTA funds is forwarded to the
Michigan State Bar Foundation. It is used to fund to support civil legal services to the poor or projects
that enhance the administration of justice.

Test: An IOLTA account shall include only client or third person funds that cannot earn income for the
client or third person in excess of the costs incurred to secure such income while the funds are held.

WARNING: The pre-mature withdrawal of trust account monies is misappropriation/embezzlement.


Embezzlement is a crime!

No commingling!

Misuse = misappropriation.

G.A. v. Frederick Sauer: Sauer moved to Hawaii with $10,000 he was to maintain in trust. Thereafter, Sauer
commingled the funds resulting in a shortfall. Holding: Sauer acted in willful disregard for his duties as a
fiduciary by depositing the funds in his general account. Two year suspension from the practice of law.

G.A. v Brent Johannsen: 1990 Cooley grad. Disbarred for misappropriating $250,000 from a client and additional
monies from colleagues. Embezzlement over $100,000 is a 20 year felony offense that also carries a $50,000 fine
or 3 times the value of the property or money embezzled, which ever is greater.

G.A. v Norman Farhat: Lansing lawyer, well respected. Unblemished record for 19 years. Stole from his clients
repeatedly to pay personal debts and repay other clients. Disbarred.

G.A. v William Conlon. “Borrowed” $28,000 to pay personal debts. Repaid the money within days.
The misappropriation was not discovered until 3 years later during a routine audit. One year suspension from the
practice of law.

G.A. v Clinton House. Allowed his secretaries to manage his trust account. Several checks were mistakenly drawn
on the account, resulting in misappropriation and conversion of funds. Although no evidence that the checks were
drawn with the respondent lawyer’s knowledge, the monthly statements were not regularly reconciled with the trust
account ledgers and the staff had had no training on how to manage the account. Holding: Whether the
commingling and conversion of funds was done as the result of some planned conscious scheme or, was the result
of careless bookkeeping, the evidence remains that respondent knew, or very certainly should have known, that he
was failing to carry out his duties as a fiduciary.

Attorneys are responsible for discipline purposes for the acts and omissions of their employees who must be
properly trained and supervised.

Trusted bookkeeper not properly supervised. Millions lost in an international fraud scheme. In 2002, sixty year-
old Ann Marie Poet, a grandmother and part-time secretary/bookkeeper for the respected Michigan law firm,
Olsman Mueller & James, thought she'd hit the big time. A fax from the "The Ministry of Mining" in South
Africa advised that her help was urgently needed to transfer money to America. In exchange, Poet would receive
a $4.5 million commission. The Hook: Various “fees” had to be paid up front, before the big payoff. These fees
were far beyond Poet’s ability to pay. No Problem! Poet, a trusted employee for nearly a decade, paid all of these
fees from Olsman’s trust account. From February to August, Poet wired amounts ranging from $9,400 to $360,000
from the law firm's account to offshore accounts in South Africa and Taiwan. In all, she “borrowed” $2.1 million –
wiping out the entire trust account.

Charles M. Zwick: Family of Wilma Southwell,files a request for Eaton County court guardian Charles M. Zwick
to account for nearly $350,000 in missing money from the deceased Lansing woman's estate. The family of Alice
Grahn - Southwell's sister - files another complaint alleging Zwick took about $285,000 from Grahn's accounts.
Judge signs a judgment against Zwick the same day. Zwick fails to appear for a show cause contempt hearing and
warrant is issued for his arrest. Zwick is arrested pleads no contest in Eaton County Circuit Court to two counts of
contempt of court for failing in his fiduciary duty to account for more than $630,000 in missing money in two
elderly Lansing's women's funds and is jailed for 60 days. The FBI investigated and the United States Attorney
charged both Zwick and his long time office manager with Federal crimes related to the scheme. At their
respective plea proceedings, both Zwick and the office administrator admitted that between 2000 and July 2005,
when the law office closed, they had regularly and systematically taken money from the accounts of the wards and
estates of clients of the practice. They admitted to having taken approximately $1.5 million in assets from nearly
100 victims. Zwick was sentenced to 60 months incarceration and 3 years supervised release. The office manager
was sentenced to 54 months’ incarceration and 3 years supervised release. In addition, both were ordered to pay a
$200 mandatory special assessment as well as to pay $1,656,408.94 in restitution.

Rule 3.4: Fairness to Opposing Party and Counsel


● A lawyer shall not:
➔ unlawfully obstruct another party's access to evidence
Washington v Olwell: Lawyer served as depository for a murder weapon.

In re Ryder: Lawyer concealed client’s sawed-off shotgun and stolen money in


his safe deposit box destroying the chain of evidence linking his client to the
contraband.

➔ alter/destroy/conceal a doc or other material having potential evidentiary value OR


counsel/assist another to do so OR
➔ falsify evidence OR
➔ counsel or assist a witness to testify falsely OR
➔ offer an inducement to a witness prohibited by law OR
➔ Kingly disobey an obligation under rules of tribunal except open refusal based on
assertion no valid obligation exists.
Example – You can not fail to pay fines/costs or to comply with a court order or
set procedure such as discovery.

You have the right to challenge an order but only through the legal process and
you must comply with a final order.

➔ In pretrial, make frivolous discovery request or fail to make reasonably diligent effort to
comply with opposing party's proper discovery request
➔ allude to any matter that the lawyer doesn't reasonable believe is relevant or that will not
be supported by admissible evidence
➔ assert personal K of the facts in issue except when testifying as a witness
➔ state personal opinion as to the justness of a cause, credibility of a witness or
culpabilityof civil litigant or guilt/innocense of an accused
Improper: “How can you believe this witness? I’ve heard a lot of people testify
over the years and in my opinion, this witness is lying. I know because he failed
a polygraph examination. I don’t believe him. How can you?”

Proper: “How can you believe this witness? His testimony is contradicted by
three other witnesses who, unlike him, have no personal interest in the outcome
of the case.”

➔ request a person other than a client refrain from voluntarily giving info to other party
UNLESS 1) person is a relative/employee/agent of client AND 2) lawyer reasonably
believes the person's interest won't be adversely affected by refraining from giving such
info.

protects important right to obtain evidence through discovery of subpeona

Many jurisdictions make it n offense to destroy material for purpose of impairing its avabilable for a
pending proceding or one who's commencement is foreseeable.

Falsifying evidence is usually a criminal offense.

A lawyer is permitted to take temporary posession for purpose of conducting a limited examination.

You may pay a witness’ reasonable expenses and reasonably compensate an expert witness for his or
her time.

Expert witness NO contingency fee.

Lay witness NO appearance fee.

Inducements for witnesses that are legal and proper


1) Paid informants
2) immunity in exchange for testimony

These inducements are considered appropriate because their purpose is not to alter testimony but to
secure it. RATIONALE: The U.S. Supreme Court has acknowledged the importance of using
immunity and other techniques to secure testimony in criminal cases.

Caveat: People v Hayes


On New Years Day 1980 Hayes, aided by James, murdered Patel. Both were charged with the
crime. James testified against Hayes and Hayes was convicted. Some 25 years later, Hayes’
conviction was overturned. Why? The Ruse: It was discovered that in order to secure James’
testimony, the prosecutor had entered into a “secret” deal with James’ lawyer. In exchange for
the testimony, James was to receive immunity in the case and other felony charges against him
would be dropped. James was not told of the deal so that at trial he could “truthfully” testify
that he wasn’t receiving any special treatment in exchange for his testimony.

LAXTON: You may not obstruct your opponent’s ability to gather evidence. Therefore, you can not
keep witnesses from the adversary. Example – Cannot buy witness an all expenses paid trip to outer
Mongolia during the discovery and trial periods.

Fact witnesses do not belong to you!

However, you can advise a witness that they don’t have to volunteer information. Unless subpoenaed,
whether a witness is interviewed is up to the witness.

Problem: they sent over photocopies that are too light to read. Send a letter to opposing counsel, ask them to
provide new copies at their cost. If they don't, file motion to compel and ask for sanctions

Problem: You're aware of new case-law and want to avoid an ex parte situation. Call judge's law clerk, ask
questions procedurally how does the court want to handle this. Prepare a memo asking to supplement the record
to include the case in the advanced sheet. Opposing counsel must be brought in even if kicking and screaming.
Set up a hearing where you can make a motion to supplement the record. Depends on local custom.

Class Action: Movie w/Gene Hackman.

Watch the wording of discovery requests. Discovery requests MUST be correct. Analogy: piano (D attny) and
piano player (P's attny).

Ok to bury opposing party if they make an overly broad request, provided you don't try to alter location of
evidence in the boxes you send over.

Witness Coaching (aka rehearsing, horse-shedding, or sandpapering)


You can interview a witness and prepare them for testimony, but you can not “suggest” that the witness
testify falsely. Telling a witness what to say (if it is false) or not to say (if to conceal a material fact) is
subornation of perjury - a felony. Drawing the Line: You have crossed the line any time you put your
own words in the witness’ mouth.

Film Clip: Intolerable Cruelty (George Clooney divorce attny)

Trial of Zacarias Moussaoui (2006): Transportation Security Administration lawyer Carla


Martin
emailed hundreds of pages of trial transcripts to 7 witnesses supplemented with her
observations, suggestions and talking points, to influence their testimony in the government’s
favor. The Court barred testimony from six of the subject witnesses –on the basis of improper
coaching. Martin’s conduct thwarted the entire purpose of sequestration: 1) to frustrate efforts
by witnesses to conform their testimony to that of earlier witnesses 2) interfering with the jury’s
ability to detect false testimony by considering varying accounts of the same events.

Discovery Example: Traxler v Ford Motor Company: 1990 rear end collision of a Ford Tempo at 50 mph. Driver’s
seat moved backwards striking a two-month old child in the head resulting in severe and permanent injuries. The
parents sued Ford for defective design of the driver’s seat. The Discovery Process:
In discovery, plaintiffs sought information on the seat design and Ford’s knowledge of the tendency of the drivers seat
to give way in rear-end collisions. The Cover Up: For more than two years, Ford, through its attorneys, concealed
significant documents and information. Lies and Games: Ford failed to disclose relevant and non-privileged
information, such as the number of lawsuits involving this particular seat design. Ford claimed there had been only
two lawsuits. In fact, there had been 91 lawsuits involving the Tempo and 100s more involving the same seat
assembly in other vehicles. Ford seldom answered interrogatories, produced documents or cooperated in depositions
without burdening opposing counsel with delays and the need to fight for everything. Every discovery request had to
be enforced through motions to compel. Ford filed groundless objections to discovery requests and when they did
answer, the answers were dishonest and carefully crafted to mislead. Holding: Anything other than the word “lied”
would understate what Ford has done. “An outrageous fraud was perpetrated by Ford.” The discovery abuses by
Ford warrant entry of a default judgment as sanction for such conduct. “It should be axiomatic that hiding
information and lying can not be tolerated and must be answered with sanctions that unmistakably say so.”

Rule 1.1: Competence


● A lawyer shall provide competent rep. and that requires the legal knowledge, skill,
thoroughness and prepartion reasonably necessary for the rep.

Factors:
1. relative complexity and specialized nature of the matter
2. lawyer's general XP
3. lawyers training and XP in field in question
4. prep or study lawyer can give to the matter
5. whether it is feasible to refer the matter to, or associate or consult with a lawyer of established
competence.

Standard: The required proficiency is usually that of a general practitioner although expertise in a
particular area of law may be required in some circumstances. (e.g., patent work).

Need not have prior XP. Lawyewr can acquire through study or associateion w/lawyer of established
competence.

In an emregency, lawyer may give advice, limited to that reasonably necessary under the
circumstances, even if the lawywer doesn't have necessary skill.

Client and lawyer can have an agreement limiting the scope of the rep.

Lawyer should keep abrest of change in the law through continuing study and education and comply
with continuing ed. requirements.

Claim Diagnosis: One of the most fundamental legal skills consists of determining what kind of legal
problem is involved. Failure to identify a meritorious claim or defense, an applicable Statute of
Limitations, or the most favorable venue for bringing a claim is malpractice and will get you grieved.

The Bottom Line: Don’t accept a matter unless you are qualified to handle it or can associate with
someone who has the requisite knowledge and expertise for the case.

Incompetence often results in neglect. Neglect = discipline and malpractice liability.

Rule 1.11: Special Conflict of Interest for Former and Current Govt Officers/Employees
● (a) – a former government attorney is subject to Rule 1.9(c) (can not use/reveal former client
info) and can not, in private practice, represent a client in a matter in which the lawyer
participated personally and substantially unless the government agency consents in writing.
Example: G.A. v Plachta – Former prosecutor handled defendant’s appeal of the very
sentence the former prosecutor achieved in the case.

● (b) – Imputation: The rule allows a former government attorney to be “screened off” (no
access, no communication, no sharing in the fee) to avoid the lawyer’s new firm from being
disqualified from cases involving the former government lawyer’s former employer. In the case
of a “screen,” written notice must be immediately given to the appropriate government agency
to enable it to ascertain compliance with this Rule.

● (c) – Use of Gov't Information: Except as law may otherwise expressly permit, a lawyer in
knowing possession of confidential government information about a person, gained when the
lawyer was a government/public officer or employee, may not represent a private client whose
interests are adverse to that person in a matter in which the information could be used to the
material disadvantage of that person.
Example: AGC lawyer that goes into private practice and sues an attorney that was
disciplined on an unrelated matter - the former AGC lawyer gained confidential
information about the attorney from the disciplinary proceeding.

● (d) – Application of Rules 1.7 and 1.9 to Government Lawyers: Government lawyers are
subject to Rules 1.7 and 1.9; and, shall not participate in a matter in which the lawyer
participated personally and substantially while in private practice or non-government
employment, unless the appropriate government agency gives informed consent, confirmed in
writing. Nor shall a lawyer, handling a case for the government, negotiate for employment with
a party or a party’s counsel while the case is pending.
EXCEPTION: A lawyer serving as a law clerk to a judge or other adjudicative officer or
arbitrator, may negotiate for private employment as permitted by Rule 1.12(b) and subject to the
conditions of that rule. (Must be with the court’s knowledge and approval).

● (e) – Definition of “Matter”: As used in this Rule, “matter” includes any judicial or other
proceeding, application, request for a ruling or other determination, contract, claim,
controversy, investigation, charge, accusation, arrest or other particular matter involving a
specific party or parties and any other matter covered by the rules of the appropriate
government agency.

Rule 7.3: Direct Contact w/Prospective Clients


● A lawyer shall not by in-person, live telephone, or real-time electronic contact solicit a
prospective client when significant motive is lawywer's pecuniary gain UNLESS person
contacted:
1. is a lawyer
2. has a family, close personal, or prior professional relationship w/lawyer.
● A lawyer shall not solicit (even folks allowed above) IF
1. prospective client lets lawyer known he desires no solicitaiton
2. solicitation involves coercion, duress or harassment.
● Every written, recorded, or electronic solicitation must include the words “Advertsing Material”
on the outside envelope OR at beginning AND ending of recorded/electornic commo UNLESS
recipient is a lawyer or family/close personal/prior prof.
● lawyer may participate w/prepaid or group legal services plan (NOT owned/directed by the
lawyer) that does solicit directly for memberships/subscriptions.

trained advocate can potentially abuse his influence.

Best to maintain a permanent record of the communications.

If lawyer makes contact and recieves no response, must not make further commo.

Don't have to mark “Adverting Material” IF commo sent in response to request by prospective client.

General announcements about changes in personnel, office location, etc. are not solicitations.

G.A. v. Michael Balian (2001) – never make the first contact

G.A. v Eric Von Weigen (1990) – use of a runner is forbidden. LAXTON: Be careful if you hire a marketing
firm, you've got to make sure what they are doing on your behalf is within the Code

Pass out handbills still direct solic. Hire someone to hand out flyers = same thing, you cannot do it.

LAXTON: Always prepare a CYA client engagement letter if there is any question about how the client contact
occurred.

The bar is not looking for ACTUAL ABUSE, it is looking for POTENTIAL ABUSE via direct solicitation.

Rule 7.3 is the codification of Ohralik and Primus.

Ohralik v Ohio Bar Assn (1978): State could have a blanket ban on face-to-face solicitation,
whether or not it is harmful in particular case. Too much risk of fraud, undue influence,
overreaching and similar evils.
Bad Facts for Ohralik
➢ He approached girls when they were vulnerable (one was still in traction at the
hospital).
➢ He urged his services upon them by “offering” to represent them.
➢ He used a concealed tape recorder.
➢ He described his fee arrangement in a “slick and tantalizing” manner.
➢ He refused to withdraw when asked to do so the next day.
➢ Lawyers are trained in art of persuasion.

The Ohio Supreme Court's indefinite suspension of the attorney was upheld by the
United States Supreme Court, which rejected his contention that his conduct was
protected by the First Amendment. Ohralik argued that state could not discipline him
unless it showed ACTUAL abuse.

Court rejected and held that a state may adopt prophylactic rules that forbid in-person
solicitation of fee generating business under circumstances that are likely to produce
fraud, undue influence, or similar evils.

In Ohralik, the Court distinguished between in-person solicitation from a newspaper ad,
finding that the former was far more coercive. “Unlike a public advertisement, which
simply provides information and leaves the recipient free to act upon it or not, in-person
solicitation may exert pressure and often demands an immediate response, without
providing an opportunity for comparison or reflection.” In person is high pressure sales
that demands immediate response and gives no time for comparison and reflection or
counter-information from another attorney.
Again, big concern over potential for fraud, undue influence, intimidation, overreaching
and other forms of vexatious conduct.

In Re Primus: Can directly solicit if no pecuniary gain sought. Political speech so must show
actual abuse/actual harm.
In Re Primus (1978): In Primus the lawyer had been invited to speak to a group of welfare mothers
who had been sterilized as a condition to continued receipt of welfare benefits. The lawyer described
to the group their legal rights and later wrote a letter to one of the attendees advising her that the
American Civil Liberties Union (ACLU) had agreed to provide representation to the sterilized
mothers without fee. While the lawyer was a cooperating lawyer with the local branch of the ACLU,
she received no compensation for her work on behalf of the ACLU. The South Carolina Bar
Association punished the lawyer for soliciting a client on behalf of the ACLU. Finding that the
lawyer's actions were "undertaken to express personal political beliefs and to advance the civil-
liberties objectives of the ACLU, rather than derive financial gain,“ the Supreme Court found that her
conduct was protected by the First Amendment rights of expression and association. Decided on the
same day as Ohralik, one year after Bates). Unlike Ohralik no pecuniary gain sought. ACLU sought
to use sterilization issue as a vehicle for political expression and association as well as
communicating useful information to public. Political Speech so State must show ACTUAL abuse.
No showing of abuse so no discipline.

Rule 7.2: Advertising


● You can advertise by any form of media.
● You can’t pay someone to recommend your services except you can pay for advertising and can
participate in a legal referral service.
● All ads must include the name and address of at least one lawyer or law firm responsible for its
content.
● You can refer clients to another lawyer or non-lawyer pursuant to reciprocal agreement but ONLY if
agreement is non-exclusive and client is informed of nature of agreement.
G.A. v Walter Finan (1993) Ida Washington had a wrongful death claim. Willie Silas, a
non-lawyer, gave Ida Walter Finan’s business card, specially recommended Finan to her
and accompanied Ida to the initial consultation. He was present when Ida signed the
contingent fee agreement. Ida’s claim was a good one. The case resulted in a $250,000
fee to Finan. Thereafter, Finan issued seven checks to Silas, totaling $3,515.00. Finan
was charged with violating Rule 7.2’s ban on paying another person to recommend your
services. Finan denied the charge, claiming that the payments to Silas were just gifts
and loans out of gratitude and because Silas was “down on his luck.” Finan, who
previously enjoyed an unblemished record, received a public reprimand.

Zauderer v Ohio: Can advertise in newspaper ads for targeted group with particular legal issue.
Ads do not invade privacy. Concerns identified in Ohralik not present. Much less risks and much less
pressure.
In Zauderer, the lawyer’s newspaper ad was directed at users of the Dalkon Shield (a birth control device).
The ad advised that many lawsuits had been filed over the I.U.D. and that it was not too late to sue. The ad
invited potential plaintiffs to contact the lawyer’s firm which was experienced in this type of claim. Ohio’s
Position: although Ohio conceded that the ad was not misleading, it found that the ad constituted improper
solicitation because of it’s targeted nature. The lawyer was disciplined. The attorney appealed – all the way
to the U.S. Supreme Court. Holding: Reversed. The concerns identified in Ohralik are not present in
targeted newspaper advertising:
-The ads do not invade privacy;
-There is much less risk of overreaching and undue influence; and,
-There is no pressure or coerciveness as exists with in-person solicitation.

Central Hudson Gas and Elec. Corp. v Public Service Commission (1980): Pursuant to Central
Hudson, while states were free to regulate commercial speech that was misleading or concerned
unlawful activity, truthful commercial speech could not be regulated (restricted) unless a 3 part test was
satisfied.

The Central Hudson Test:


For a restriction on truthful commercial speech to withstand scrutiny, a state must be able to
demonstrate that:
1) The asserted government interest behind the restriction is substantial;
2) The regulation directly advances that interest; and,
3) The regulation is not any more extensive than necessary to serve that interest.

Effect of Central Hudson Today: Central Hudson continues as the current first amendment standard
for evaluating restrictions on commercial speech.

RMJ (1982): The 1st lawyer advertising case to apply the Central Hudson test. The restriction under
scrutiny in RMJ required that a lawyer use specific terminology in describing areas of practice in an
advertisement and also required a disclaimer regarding competency in any given area of practice. In
RMJ, the lawyer had failed to use the required terminology and had not included a disclaimer. The
lawyer was disciplined. On appeal, the restriction was stricken down as unconstitutional under Central
Hudson. The discipline was vacated.

Shapero v Kentucky Bar: Can send targeted mail to individuals with particular legal issues.
Absence of risk factors of Ohralik. Cannot ban any activity short of direct solicitation if advertising is
truthful and not misleading unless the restriction satisfies the three prong Central Hudson test.
Dissent on pg. 99: O'Connor argued that Bates should be reconsideredd and that allowing attnys to hawk their
wares will eventually destroy the professionalism that limits the “unique power” attnys wield in our political
system. States should have “considerable latitude” to ban advertising...even truthful advertising that
undermines govt interest in promoting high ethical standards in legal profession.

Shapero’s Aftermath
After Shapero, states were unable to ban any activity short of direct contact solicitation unless the
restriction satisfied the three prongs of Central Hudson
Went For It Case: State regulation upheld because restriction satisfied Central Hudson test.
Significant state interest in protecting public from invasive conduct of lawyers and preserving the
reputation of the legal profession; The restriction directly and materially advanced these interests; and
The restriction – just 30 days – was narrowly drawn.
Facts: a state restriction on the timing of letters to accident victims came under scrutiny. Pursuant to the
restriction, lawyers were prohibited from sending letters soliciting accident victims within the first 30 days
after the accident. A lawyer in violation of the restriction, challenged its constitutionality. Restriction
Upheld by the U.S. Supreme Court, holding that the 30 day restriction met the three prongs of Central
Hudson. Reasoning: Florida had demonstrated a significant state interest (protecting the public from
invasive conduct of lawyers and preserving the reputation of the legal profession). The restriction directly
and materially advanced these interests. The restriction – just 30 days – was narrowly drawn.

Coupons
States are split on the issue of coupons. Alabama, Connecticut, Pennsylvania and Michigan have ruled
that they are permissible. Ohio has ruled that they are not.

Consumer Fraud Laws


Beware: Some states, Colorado among them, have held that deceptive lawyer advertising gives rise to
a civil cause of action under the state’s consumer protection laws. Crowe v. Tull, 126 P.3d 196 (Colo.
2006). (Colorado Consumer Protection Act held to apply to allegedly deceptive trade practices by
attorneys.)
Rationale: When professionals start conducting their activities in a way more like commerce,
such as engaging in advertising, they should be subject to laws that govern commercial
activities. Consumers are entitled to the same protection.

Opposite View: New Jersey. The CPA has never been applied to lawyers and other professionals.

Rule 1.3: Diligence


● A lawyer shall act w/reasonable diligence and promptness in representig a client.
Should pursue matter depsite opposition, obstruction or personal inconvenience and have zeal in
advocacy on client's behalf.
Not bound to press for every advantage.

Should control work load so each matter can be handled competently.

Avoid procrastination as can adversely affect case, change conditions (new law) or overlook statute of
limitations. Also creates anxiety in client and undermines attny-client relationship.

Reasonable requests for postponement are ok.

Lawyer should clarify whehter attny-client relatioship still exists/when it ends preferably in writing.
Should consult w/client about possiblity of appeal before reqlinquishing response.

Sole practicioners should have plan in case of death or disabliity that designates another competent
lawywer to review client files and notify them of death/disability of attny.
Rule 3.8: Special Responsibilities of a Prosector
● Prosecutor shall:
● refrain from prosecutin charge P K's isn't supported by probable cause
NOTE: This is a stricter standard than the non-frivolous standard in Rule 3.1. A
criminal charge must be more than non-frivolous. It must be supported by
probable cause.
● make reasonable efforst to assure the accused has been advised of right/proc for
obtaining counsel and reasonable opportunity to do so.
● Not seek to obtain from unrep'd accused a waiver of important pretrial rights like prelim.
hearing. (Doesn't apply if D is pro se)
● Make timely disclose to defense of all evidence/info that tends to negate guilt of accused
or mitigate the offense. In connection w/sentenceing, disclose to D and tribunal all
unpriv'd mitigating info EXCEPT when the P is relieved of this responsibility by
trbunal.
● Not subpeona a lawyer in grand jury or other proc to present evidence about past/present
client UNLESS P reasonably believes:
1. The info sought is not protect from disclosure by priv; AND
2. evidence sought is essential to success of ongoing prosecution; AND
3. no other feasable alternative to obtain info.

To limit the issuance of lawyer subpoenas to situations where there is a genuine


need to intrude upon the attorney-client relationship. A prosecutor must have
court approval for the issuance of a subpoena after opportunity for an evidentiary
hearing on the issue of need.

● refrain from making extrajudicial statements that have a substantial likelihood of


heightening public condemnation of accused AND exercize reasonable care to make
sure investigates law enforcement, etc. don't make statements P is prohibited from
making.

The prosecutor is not just an advocate. A prosecutor is a minister of justice. This carries with it special
responsibilities of ensuring due process in the proceedings. The duty is to seek justice not merely to
convict.

Rule 3.8(d) is the Codification of Brady v Maryland (U.S. Sup Ct, 1963)
Brady involved the post-sentencing discovery that the prosecutor had withheld a statement by an
accomplice confessing to the murder for which the defendant was convicted.

The Brady Duty:


● The prosecutor must inform the defendant of evidence that tends to negate guilt or mitigate
punishment.
● The prosecutor must turn over evidence that can be used to impeach government witnesses.
● The prosecutor must follow through on leads regarding possible exculpatory evidence.
● The prosecutor must turn over exculpatory information to the defense even in the absence of a
request. United States v Agurs

Policy Considerations: Convictions should rest only on accurate evidence and on a record that is as
complete as possible on the question of innocence.
The Disregard for Brady: A 2003 review of appellate decisions of cases coming out of the Bronx
revealed prosecutors having been cited for misconduct in 72 cases over a 21 year period. Many of the
cases involved Brady violations. The misconduct was egregious and resulted in reversals in 62 of the
cases.

People v Alberto Ramos (1992): Ramos was 21 years old and a part time employee at a day care center in the
Bronx when he was arrested and charged with the rape of a five year old girl at the center.
Ramos was convicted and while in prison endured beatings, was brutally sodomized every day, and tried to
commit suicide several times. In 2003, Ramos won a $5 million settlement 11 years after he was released
from prison because of a wrongful conviction in the case. Holding: The prosecutor withheld evidence that
likely would have exonerated Ramos. What the jury never heard was testimony from the day care center
workers that the girl had masturbated frequently in class, had extensive knowledge of sexual acts and had
used sexual language prior to Ramos working there. Turns out that she had been exposed to pornographic
movies. They also never heard testimony that the child had told city social workers that Ramos had not done
anything to her. Ramos indicated he would give back every penny if he could turn back the clock and not
have to endure prison. The prosecutor, Diana Farrell, had a duty to make this evidence known to the defense
but failed to do so. Guess it just didn’t “fit in” with her case.

Tony Flores (former prosecutor)


A homicide witness often recants. Domestic assault victim often recants (about ½ the time). Technically
under BRADY prosecutor has to disclose to D attny. Doesn't matter if you believe victim or not, fact that
victim recanted is Brady material. (MA holds that way)
Prosecutor imputed with knowledge of law enforcement, or support (social workers)
If recanted, need to memorialize by getting a new statement from the victim

Other examples: Duke Rape Case and Nifong's disbarment. People v. Martha Stewart: The prosecutor had
failed to turn over an FBI memo that might have cast doubt on the credibility of the government’s key
witness. The prosecution was penalized by not being allowed to call this witness, Merrill Lynch broker
assistant Douglas Faneuil, until later in the case. (not much of a penalty) One week later, nearly a dozen
more such “smoking gun” documents surfaced.

Rule 7.1: Communications Concerning a Lawyer's Services


● A lawyer shall not make false or misleading communication about the lawyer or his services. A
commo is false if it is a material misrep of fact or law or omits facts necessary so statement isn't
materially misleading.

Rule 7.1 is the codification of Bates.


Bates v Arizona – US SC: Advertising ban was violation of free speech if advertisement was
truthful. Advertising that is false, deceptive or misleading can be restrained.
Bates and O’Steen started their own legal clinic in Phoenix. They concentrated on providing low-
cost legal services to the poor. The problem: The need for a steady stream of clients to keep the
business afloat. The Solution: “After two years, we concluded that the clinic would not succeed if
we did not advertise.” On Feb. 22, 1976, Bates & O’Steen took out an ad in The Arizona Republic.
The ad, which was truthful, listed prices for uncontested divorces, uncontested adoptions, personal
bankruptcies and name changes. Arizona had a total ban on lawyer advertising. As soon as Bates
and O’Steen placed the ad, they called their former Con Law Professor, Bill Canby. Arizona handed
down a suspension. Bates & O’Sheen appealed. At US SC Professor Canby pressed his First
Amendment and antitrust claims before the justices. The Arizona bar responded with a number of
arguments as to why the legal profession should be able to prohibit advertisements.

Govern's ALL communications including advertising.


A statement is misleading if is would lead a RP to form an unjust expectation.

Inclusion of appropriate disclaimer may help preclude finding statement is misleading.

Hypo: ad that says you've never lost a trial, but you've never tried a case either. That's misrep

If false or misleading->state can regulate Laxton: remember UNTRUTHFUL speech of any kind CAN be
regulated.
If not, then central hudson 3 prong applies

Chandler & Pope: “1 (800) Pit-Bull” and use of a logo with an image of the spike-collared dog.
Florida has rules that prohibit lawyers from making statements: “describing or characterizing the quality of
the lawyer’s services in advertising and written communications;” and, forbid “visual or verbal descriptions”
that are not objectively relevant to the selection of an attorney” or are “deceptive, misleading or
manipulative.” Pit Bull Symbolism: Pape & Chandler argued that the ads we meant to promote the positive
characteristics associated with a Pit Bull Terrier: strength, loyalty, courage and tenacity. The Florida
Supreme Court rejected this notion: “We consider this a charitable set of associations that ignores the darker
side of the qualities often also associated with pit bulls: malevolence, viciousness and unpredictability.”
Florida Muzzles Pit Bull Ads: In December 2005, the Florida Supreme Court found that the ads were
unethical. The lawyers received a reprimand.
Holding: The ads “demean all lawyers and thereby harm both the legal profession and the public’s trust and
confidence in our system of justice.” “Lawyer advertising enjoys First Amendment protection only the
extent that it provides accurate factual information that can be objectively verified.” The ads are both
manipulative and misleading. “We can not condone an ad implying lawyers will gets results through
combative and vicious tactics that will main, scar or harm the opposing party. If we approved the ad, images
of sharks, wolves, crocodiles and piranhas could follow.”

Rule 4.2: Communication w/Person Represented by Counsel


● In representing a client, lawyer shall not communicate about the subject of the rep with person
lawyer Ks to be rep'd by counsel UNLESS the lawyer has consent of other lawyer or is auth'd to
do so by law or court order.

Protects against overreaching and interferring w/attny-client relationship.

Rule applies even though the rep'd person initiates or consents to the comunication.

Lawyer must immediately terminate communication if lawyer finds out (subsequently) that other
person is rep'd.

Lawyer cannot make the prohibited communication via another person.

Parties to a matter, however, MAY communication w/each other.

Communications may be auth'd by law such as investigative activites of lawyers reprenting govt.
entites. When communicating w/accused, govt. lawyer MUST comply with this rule.

A lawyer who isn't sure commo is auth'd may seek a court order in exceptional circumstances. For
example, to avoid reasonably certain injury.

Consent of an org's lawyer isn't required for communication w/former employee but lawyer cannot use
methods ot obtain evidence that violate the legal rights of the org.

Lawyer must have ACTUAL K can be inferred from the circumstances. Lawyer cannot close eyes to
the obvious.

It depends on whether the opposing party is known to be represented by counsel. If you know that the
party has a lawyer, such direct contacts are improper.

G.A. v George Ashford (1995) (Defense counsel engaged in an intimate relationship with a class
action plaintiff, communicating with her about the case, without her lawyer’s knowledge or
consent). Reprimand with conditions.

G.A. v Hindelang (1990) (Lawyer for defendant met with the plaintiff without notice other
counsel, gave the party a letter for her signature to present to her attorney. The letter stated that
the plaintiff did not want to pursue the litigation, instructed plaintiff's attorney to prepare
documents dismissing the litigation, and discharged him from the case). Reprimand.

G.A. v Michael (1991) (Lawyer communicated directly with the opposing party about a divorce
settlement proposal). Reprimand.

G.A. v Kwyer (2000) (Lawyer communicated with the opposing party in a divorce through his
client). One year suspension.

Where the Client is an Organization: An organization is made up of its constituents (employees).


The Rule extends only to:
1) Employees with managerial responsibility – those that direct and supervise or who regularly consult
with the organization’s lawyer about the matter; and,
2) Employees whose acts or omissions may impute liability (civil or criminal) to the company; and,
3) Employees whose statements may constitute admissions on the part of the organization.

Direct contact is permitted with:


1) Low level employees whose conduct or scope of employment isn’t involved in the disputed
events
AND
2) Former officers and employees even if they were in one of the categories under which
communication was prohibited while they were employed. (No longer an agent of the company – now
a fact witness not owned by anyone).

LAXTON Practice Tip: Document ALL communication with pro per counsel

Rule 1.13: Organization as Client


● If a lawyer for an organization knows that an officer, employee or other person associated with
the organization is engaged in action, intends to act or refuses to act in a matter related to the
representation that is a violation of a legal obligation to the organization and that is likely to
result in substantial injury to the organization, then the lawyer shall proceed as is reasonably
necessary in the best interest of the organization.
● Unless the lawyer reasonably believes that it is not necessary in the best interest of the
organization to do so, the lawyer shall refer the matter to higher authority in the organization,
including, if warranted by the circumstances to the highest authority that can act on behalf of
the organization as determined by applicable law.

Note: Depending on the position of the clients with the company it could result in a shareholder
derivative lawsuit if it weakens the stock

The Sarbanes-Oxley Act of 2002


Sarbanes Oxley was drafted by Congress in the aftermath of the financial collapses of Enron,
WorldCom, and Global Crossing. The Act, which became effective in July 2002, established a new
regime of accountability by public companies in the areas of financial reporting, disclosure, audits,
conflicts and governance.

Under Sarbanes, attorneys and accountants are more accountable. They must report evidence of
material violations of federal securities laws or breaches of fiduciary duties to appropriate company
officials.

This was a big change from the “wink and nod” approach to corporate governance of the 1990s. In that
era, lawyers and accountants were protected from civil liability for corporate fraud. Central Bank of
Denver v First Interstate Bank of Denver, 511 US 164 (1994) had eliminated private actions against
such secondary actors for aiding and abetting corporate wrongdoing. This is no longer the case.

Section 307 governs lawyers.


● Requires a lawyer to report evidence of material violations of federal securities laws or breaches
of fiduciary duty to the general counsel or chief executive counsel of a company the lawyer
represents.
● If the general counsel or CEO does not appropriately respond to the information, the lawyer
must report the matter to the company’s board of directors or audit board.
● The lawyer who reports the evidence “up the ladder” without appropriate response must make a
“noisy withdrawal” from representation. This “noisy withdrawal” must include a written
disavowal to the SEC of any document the lawyer helped prepare and file with the SEC
containing questionable information.

The Conflict Between Sarbanes and the Rules


The “up the ladder” reporting requirements of Sarbanes apply to all lawyers who represent public
companies. These requirements, however, at the time of enactment, directly conflicted with the Rules.
Specifically, the rules governing confidentiality did not permit such disclosures. The New Model Rule
1.13 Organizations as Clients fixed that. Two subsections set up an “exhaustion of internal remedies”
scheme similar to the “up the ladder” reporting requirements of Sarbanes. Through these subsections,
the RPC are now consistent with the requirements of Sarbanes.

Rule 1.13(a):
A lawyer employed or retained by an organization
represents the organization
acting through its duly authorized constituents.
Pursuant to Rule 1.13(a), you owe your allegiance to the entity as opposed to its individual
constituents. In other words, you must always act in the best interests of the organization as opposed to
those individual persons.

Rule 1.13(b) – The General Premise: If a lawyer for an organization knows that an officer, employee
or other person associated with the organization is engaged in action, intends to act, or refuses to act in
a matter related to the representation that is a violation of a legal obligation to the organization, or a
violation of law which reasonably might be imputed to the organization and is likely to result in
substantial injury to the organization, the lawyer shall proceed as is reasonably necessary in the best
interest of the organization.

Substantial Injury: What is substantial? All crimes? Only felonies? Are relatively minor crimes
included? The Rule does not make all crimes a per se violation.
Best measure: will there be a substantial injury if the violation is discovered?

Exhaustion of Internal Remedies Scheme:


In a 1.13(a) situation, a lawyer shall take remedial measures including, if necessary:
1) Asking for reconsideration of the matter (remonstrate with the offender);
2) Advising that a separate legal opinion be obtained; and,
3) Referring the matter to a higher authority in the organization including, if warranted by the
circumstances, to the highest authority that can act on behalf of the organization as determined by
applicable law.

Any measures taken must be designed to minimize disruption in the organization and the risk of
revealing confidential information to persons outside the organization.

Up-The-Ladder – Last Resort


If, despite the lawyer’s best efforts in accordance with paragraph (b), the highest authority that can act
insists upon an action or a refusal to act, that is clearly a violation and the lawyer is reasonably certain
the conduct will result in substantial injury to the organization, the lawyer may reveal information
related to the representation whether or not protected by Rule 1.6 but only to the extent necessary to
prevent the substantial harm/injury.

Note: Any measures taken must be designed to minimize disruption in the organization and the risk of
revealing confidential information to persons outside the organization.

Rule 1.13(d) – Exception to Paragraph (c): Paragraph (c) does not apply with respect to information
relating to a lawyer’s representation of an organization to investigate an alleged violation of law, or to
defend the organization or an officer, employee or other constituent associated with the organization
against a claim arising out of an alleged violation of law.

Policy Reason for Exception: To enable organization clients to enjoy the full benefits of legal counsel
in conducting an investigation or defending against a claim.
Rule 1.13(e)

A lawyer who reasonably believes he’s been discharged because of the lawyer’s actions under
paragraphs (b) and (c), or who withdraws under circumstances that require or permit such withdrawal
under (b) or (c), shall proceed as the lawyer reasonably believes necessary to assure that the
organization’s highest authority is informed of the lawyer’s discharge or withdrawal.
Rule 1.13(f)

In dealing with an organization’s directors, officers, employees, members, shareholders or other


constituents, a lawyer shall explain the identity of the client when the lawyer knows or reasonably
should know that the organization’s interests are adverse to those of the constituents with whom the
lawyer is dealing.

Rationale: A problem arises when constituents are of the impression that you are representing them as
individuals in addition to the business enterprise. Example: The case of closely-held (such as family
owned) corporations.
Rule 1.13(g)

A lawyer representing an organization may also represent any of its directors, officers, employees,
members, shareholders, or other constituents, subject to the provisions of Rule 1.7.

If the organization’s consent to the dual representation is required by Rule 1.7, the consent shall be
given by an appropriate official of the organization other than the individual who is to be represented or
it may be given by the shareholders.

Rule 5.1: Responsibilities of Partners, Managers, and Supervisory Lawyers


● A partner OR other lawyer w/comparable managerial auth shall make reaosnable efforst to
ensure firm has measures that give reaosnable assurance all lawyers in firm will conform with
Rules. [shareholders too!]
● A lawyer having direct supervisory auth shall make reasonable efforst to ensure that his
subordinates conform to the Rules.
● A lawyer shall be responsible for other lawyer's violation IF:
● the lawyer orders, or with K of the specific conduct, ratifies the conduct OR
● the lawyer is a partner/managing lawyer/direct supervisor over the person and Ks of the
conduct at time when its consequences can be avoided or mitigated but fails to take
remedial action.

Examples: policies and procs to detect and resolve conflicts of interest, identify dates by which actions
must be taken, account for client funds, properly supervise inexperienced lawyers.

Small firm of exp'd lawyer inform supervision and periodic review usually suffices.

Large firm requires more elaborate measures.

Firms may also relay on continuing leal ed in prof. ethics.

Whether lawyer has supervisory auth is a question of fact.

If subordinate screws up, both sub and supervisor have duty to correct.

Whether a lawyer is responsible civilly or criminally for other lawyer is a question of law beyond scope
of the Rules.

Rule 3.1: Meritorious Claims and Contentions


● lawyer shall not bring or defend a proceeding, or assert or controvert and issue UNLESS there
is a basis in law and fact that is NOT frivolous. This includes a good faither argument for an
extension, modification, or reversal of existing law. D in criminal proceeding can require that
every element of the case is established.
No frivolous merely b/c facts have not been fully substantiated or b/c lawyer expects to develop vital
evidence only by discovery.

If law in another jurisdiction has already changed, that would be a basis.

Rule 11 Sanctions
If a frivolous claim or defense is pursued in federal court, the lawyer and party will be subject to Rule
11 sanctions. Rule 11 requires that every pleading or other paper be signed. Signature: certifies that to
the best of the submitter’s knowledge and belief after reasonable inquiry, the matters set forth in the
document are founded in fact and have not been advanced for any improper purpose.
Morbeck v Kirlan Venture Capital (Wash., 2003)
Eight out of 18 claims made in a counterclaim were without merit. Offending counsel was ordered to
pay $200,000 in Rule 11 sanctions to each of the two defendants. The lawyer’s client was ordered to
pay $50,000 to each defendant. Holding: “It is not sufficient to search for a way to do the bidding of an
influential client. It is unprofessional to accept at face value [a client’s] emotional claims. Attorneys
must be prepared to conduct an educated and enlightened analysis and to say ‘no’ to their clients when
the circumstances so dictate.”

Malicious Prosecution
“We’ve been sued for bringing a frivolous lawsuit.”

The Elements of a Malicious Prosecution Claim:


1. Initiation or continuation of the underlying action;
2. Lack of probable cause;
3. Malice; and,
4. Favorable termination of the underlying cause of action.

Probable Cause
Majority view: Objective standard – Would a reasonable attorney have pursued the action?
Minority view: Subjective standard – Did the lawyer know that the claim was frivolous?

Malice can be established by:


1. Proof of ill will; or,
2. Proof claim pursued for an improper purpose; or,
3. The lack of probable cause.

Zamos v Stroud, (Calf., 2003): Holding: Attorneys are liable for malicious prosecution if they pursue a
suit after discovery reveals no viable claim.

Rule 5.3: Responsibilities regarding Non-lawyer Assistants


● A partner/lawyer who manages non-lawyers shall make reasonable efforts to ensure that there
are measures in effect to keep non-lawyers from violating the Rules AND make reasonable
efforts to ensure that the person's conduct is compatible with the Rules.
● A lawyer is responsible for the non-lawyer's conduct IF:
● the lawyer orders, or with K of the specific conduct, ratifies the conduct OR
● the lawyer is a partner/managing lawyer/direct supervisor over the person and Ks of the
conduct at time when its consequences can be avoided or mitigated but fails to take
remedial action.

applies to secretaries investigators, law students, paraprofessionals whether or not they are independent
contractors.

Particularly important they are instructed not to disclose.

Rule 4.1: Truthfulness in Statements to Others


● While representing a client, a lawyer shall not KNOWINGLY make a false statement of
material fact or law to a third person; OR fail to disclose a material fact when disclosure is
necessary to avoid assisting in crim/fraud by client UNLESS disclosure prohibited by 1.6.

No affirmative duty to inform opposing party of relevant facts.

misrep by omission is a danger

nondisclosure of principal is permissible if it isn't fraudulent.

Settlement Negotiations and “Puffery”


You have some leeway in what you say in the course of settlement negotiations. It is recognized that
during negotiations, a party and its counsel might exaggerate or emphasize the strengths, and minimize
or de-emphasize the weaknesses, of it’s factual or legal position. Such remarks, often characterized as
‘posturing’ or ‘puffing,’ are statements upon which parties to a negotiation ordinarily would not be
expected to justifiably rely, and are distinguished from false statements of material fact.

Statements that downplay a client’s willingness to compromise, that present a bargaining position
without disclosing the client’s bottom line, or that overstate or understate the strength or weakness of a
client’s position, are ordinarily not considered ‘false statements of material fact’ within the meaning of
the model rules.

Don’t Step Over the Line!


Example #1: It would be a false statement of material fact if a lawyer representing a company in labor
negotiations told the union that a particular employee benefit would cost the employer an additional
$100 per employee when the benefit actually would cost only $20 more.

Example #2: It would be a false statement of material fact for a lawyer negotiating for a plaintiff in a
personal injury case to conceal the client’s death from opposing counsel.

Rule 3.5: Impartiality and Decorum of the Tribunal


● A lawyer shall not:
-seek to influence judge, juror, prospective juror by means prohibited by law;

In Re Warlick (SC 1985): Lawyer’s investigator had personal contact with


several prospective jurors and their families. The lawyer then selected three of
those contacted to sit on the jury. Holding: Disbarred. Where contact with
prospective jurors is informed or intentional, a strong sanction is required.
Caveat: Be certain to instruct your client to stay away from the jury.

A True Story:
A former sales director of a famous New York modeling agency sued the agency
and one of its executives. In the case, the plaintiff claimed that cigarette smoke
in the workplace had aggravated her asthma and that when she complained, she
was fired. Improper Contact: The jury found for the plaintiff on the issue of
liability. While the jury was deliberating over damages, the defendant executive
disparaged the plaintiff to a member of the jury during a chance encounter in the
restroom. “You’ll Never Believe What Just Happened!” The juror told another
juror who reported the incident to the judge. Both jurors were dismissed and
alternates were seated. The Verdict: The jury returned a $5.2 Million verdict for
the plaintiff. The executive? She was charged with Tampering with a Jury.
(NYLJ Jan. 7, 2004).

-communicate ex parte w/such person during the proceding unless authorized by law or
court order
Beware! The improper ex parte communication may be initiated by the court!

Donaldson v Donaldson: The Trial Court directed each party to simultaneously


submit written arguments on two issues without exchanging them with each
other. Holding: This procedure constituted an improper ex parte communication.
Why? Because it involved each party communicating privately with the court
without the other being informed of what was being communicated.

-communicate w/juror or prospecive juror after discharge of the jury if


1) the communication prohibited by law or court order;
2) the juror has made known a desire not to communicate;
3) the communication involves misrep, coercion, duress, or harassment;

You CANNOT communicate with the juror. Take the stairs. Do not end up in an
elevator with the juror. Talk to opposing counsel, then go talk to the judge and have the
judge remind them not to communicate with you. Technically you cannot nod your
head.

You cannot ask juror about deliberations (under fed rules) but judge can ask if the
transcript was delivered in its entirety.

-engage in conduct intended to disrupt the tribunal.


applies to any proceeding INCLUDING deposition
lawyer should not sink to same level if judge is being abusive

Matter of Vincenti: Facts: Attorney Vincenti represented a defendant in a child


abuse case. In court, Vincenti was sarcastic, disrespectful, and occasionally
irrational. In open court, he called the deputy attorney general a “bald-faced liar”
and a “thief, liar and a cheat.” Out of court he called her by cruder names. He
wrote a smarmy letter to the trial judge, essentially accusing the judge of
incompetence or corruption or both. Finally, he was rude and obscene to
witnesses, potential witnesses, opposing counsel, and other attorneys around the
courthouse. Issue: Was Vincenti’s conduct merely an excusable excess of zeal in
advocacy on behalf of his client, or was his conduct so bizarre and outrageous as
to warrant discipline? Held: Vincenti’s conduct warrants strong discipline. He is
suspended from practice for one year and until further order of the court. This
case makes a useful contrast with the Gentile case. Reasonable persons might
differ about whether lawyer Gentile’s conduct warranted discipline, but there is
no question in this matter.

Avoiding Even the Appearance of Impropriety: In September 2003, the U.S. Supreme
Court became involved in a case that concerned V.P. Cheney’s energy task force. Two
months later, and while the case was pending, Justice Scalia and Cheney, long-time
friends, along with defense secretary Rumsfeld and others, had dinner together – lamb
and crab cakes on Maryland’s eastern shore. What Looks Like a Duck…One month
later, and again, while the case was pending, Cheney and Scalia spent several days in
southern Louisiana, duck hunting together.…Quacks Like a Duck. These outings gave
rise to the appearance of a tainted process where decisions are not made on the merits
but rather on the basis of fraternizing with people before the court. …Is Probably a
Duck. Many called for Scalia’s disqualification from the case – a cry he ignored. Even
a Whiff of Smoke…As one expert put it, “There’s the old adage of where there’s smoke,
there’s fire.” “There may be no fire in this case but judges, especially U.S. Supreme
Court judges, want to avoid even a whiff of smoke.” Quid Pro Quack:
Judges need to think before they act.

That is not to say that every judge has to be a hermit. But judges are in a special position
of trust, and they need to conduct themselves in a way that doesn't raise an issue in
reasonable people's minds.

Rule 5.2: Responsibilities of a Subordinate Lawyer


● A lawyer is bound by the Rules even if being supervised.
● A lawyer doesn't violate the rules if the lawyer follows supervior's reasonable resolution of an
arguable question of professional duty under the Rules.

MINOR RULES (<10%)


Rule 8.1: Bar Admission And Disciplinary Matters
● An applicant for admission to the bar, or a lawyer in connection with a bar admission
application or in connection with a disciplinary matter, shall not:
● knowingly make a false statement of material fact; or
● fail to disclose a fact necessary to correct a misapprehension known by the person to have
arisen in the matter, or knowingly fail to respond to a lawful demand for information from an
admissions or disciplinary authority, except that this rule does not require disclosure of
information otherwise protected by Rule 1.6.

The Schware Case – 20 years previously communist party, illegal strike, alias. No evidence of
CURRENT lack of characer. Rational Connection Test: Whether PAST CONDUCT suggest
PRESENT character traits that will result in FUTURE conduct incompatible with becoming an attny
The more remote the acts, the less likely the conduct can serve as a basis for denial of admission.

Hale v. Illinois Commission on Character & Fitness 1999. Member of World Church of the
Creator. Applying the rational connection test court held that Hale had a “gross deficiency in
moral character.” 2 to 1 vote, dissent Heiple: holding a belief doesn't matter, what matters is
whether applicant will adhear to his oath appealed to US SC: did not grant certiorari

In Re DeBartolo: high school residence, impersonate police officer, 200-400 tickets

Regulation of the Bar


State Supreme Court regulates lawyers
Federal Courts may intervene if there is a “federal question”
Do I have to join the Bar? In MI it is mandatory.

Requirements for Fed Court Admission


US District Court: upon motion if admitted in good standing in a state court
US Court of Appeals: Same
US Supreme Court: same + 3 years

Special Admission Proc:


pro hac vice - “for this turn only”
file a motion for admittance pro hoc vice and align themselves with a local attny as co-counsel
law student practice rules

Character & fitness


“current good moral character” applicant's burden of proof: C&C . You need documentation.

Grievance Procedure
The filing of a grievance or Request for Investigation. Anyone can file against you. There are no
standing requirements. There is no statute of limitations on the filing of a grievance.

Duty to Answer
The rules require cooperation with a disciplinary agency.
If a grievance is filed against you, you MUST timely answer the complaint.
The answer must fully and fairly disclose the facts and circumstances related to the allegations.
Failure to answer will result in discipline.

Action After Investigation


Dismissal.
Admonishment (private warning).
Issuance of a Formal Complaint (a public charge of professional misconduct).

Formal Proceedings
Commenced through the filing of a Formal Complaint:
Notice of Judgment of Conviction; or,
Notice of Entry of an Order of Discipline in another state.

Proceedings are public.


Right to a trial on the charges, except in the instance of an out-of-state order of discipline.
The disciplinary agency must establish the charges by a “preponderance of the evidence.”

Penalties if Charges Established


Reprimand.
Probation.
Suspension.
Disbarment.
Permanent Disbarment (where valid).
These notices will go in the Michigan Bar Journal for all to see!!

Appeals
Right to Appeal to an appellate level panel.
Leave to Appeal to the state’s high court.
Leave to Appeal to the U.S. Supreme Court.

Rule 8.3: Reporting Professional Misconduct


● A lawyer who knows that another lawyer has committed a violation of the Rules of Professional
Conduct that raises a substantial question as to that lawyer's honesty, trustworthiness or fitness
as a lawyer in other respects, shall inform the appropriate professional authority.
● A lawyer who knows that a judge has committed a violation of applicable rules of judicial
conduct that raises a substantial question as to the judge's fitness for office shall inform the
appropriate authority
● This Rule does not require disclosure of information otherwise protected by Rule 1.6 or
information gained by a lawyer or judge while participating in an approved lawyers assistance
program.

Rule 3.6: Trial Publicity


● As a lawyer in a case, you shall not make an extrajudicial statement, that you know or
reasonably should know will be publicized; and, will have a substantial likelihood of materially
prejudicing the proceedings.
LAXTON STRESSED: “substantial likelihood of materially prejudicing that proceeding” test

G.A. v Riley (1997): During trial of a criminal case, the court disallowed one newly
found prosecution witness and an expert offered by the prosecution on the child abuse
syndrome. While the jury was deliberating, the prosecutor complained about the rulings
to a local newspaper. He stated, among other things, that the “cards were stacked in
favor of defendants” and that “the system should be more victim oriented.” Extra-Extra:
Read All About It! At the time of the comments, the prosecutor anticipated the return of
the jury and conclusion of the case before publication of any newspaper article. (In his
10 years as a prosecutor, he had never had a jury stay out longer than one day).
Unfortunately, the jury stayed out longer than anticipated and the article was published
before completion of their deliberations. Holding: Lack of intent to prejudice is not a
defense. Given the nature of the comments and high profile nature of the case,
Respondent’s conduct was inherently unreasonable. Respondent’s gamble that the case
would be concluded prior to publication of his statements was reckless and it is a gamble
he lost. PUBLIC REPRIMAND.

G.A. v Leodis Elliott (1995) Elliott was appointed to represent an individual charged with Assault
with Intent to Commit Murder and Felonious Assault. Five months into the representation, Elliott
gave an interview about the case. He commented, “Every time he [the defendant] gets out, he goes
and messes over someone else. I can see a clear pattern of escalating violence.” Holding: One Year
Suspension.

G.A. v O’Dette (2000): O’Dette represented Jack Kevorkian in legal matters ancillary to the assisted
suicide cases. While representing Kevorkian on a charge of Resisting and Obstructing Arrest,
Kevorkian “harvested” the kidneys of a man whose life he had helped to end. O’Dette moved to
withdraw from Kevorkian’s representation and in a media interview at the time, stated: “After the
organ deal, my seven year old son told me, ‘Dad, you shouldn’t represent him. They should throw
him in jail and melt the key.’ You know – I’ve got a family to think about here. That kind of did it
for me. [Kevorkian] went too far this time and I want no part of it anymore.” Holding: PUBLIC
REPRIMAND

● Notwithstanding rule above, you may state:


Benign facts.
Public record. (or news report, or online)
Procedure.
Warning of danger if reasonable likelihood of substantial harm

● The Right to Mitigate: You may make a statement you reasonably believe is required to protect
your client from the substantial undue prejudicial effect of recent publicity not initiated by you
or your client.
Caveat: The statement must be limited to that information necessary to mitigate the
recent adverse publicity.
Fight fire with fire provision

● Imputation: No lawyer associated in a firm or government agency with a lawyer subject to Rule
3.6(a) shall make a statement prohibited by paragraph (a). In other words, you can not
circumvent Rule 3.6 through a third person.

LAXTON: Rule 3.6 limits what you can say /trial publicity/most of the time the best comment is “no comment”
don't tip your hand about what you're going to do the next day in court.

Sensitivity to Extra-Judicial Speech


Nature of the proceeding is a factor in evaluating prejudice:
Criminal jury trials are the most sensitive to extrajudicial speech.
Civil trials may be less sensitive.
Non-jury or administrative hearings may be even less affected.

Gentile v State Bar of Nevada


Gentile represented a criminal defendant. Defendant was charged with stealing drugs and money used in an
undercover operation conducted by the police. The trial attracted a great deal of publicity. During a pre-trial press
conference, Gentile made several inflammatory statements, accusing the detective of actually stealing the drugs and
travelers checks and framing his client. Six months letter, a jury acquitted the defendant on all charges. State Bar filed
a complaint alleging a violation of a rule very similar to MR 3.6 Rule prohibited attorneys from making “extrajudicial
statement” that a reasonable person would expect to be spread among the public and to materially prejudice the
proceedings. Discipline board found a violation but SC reversed on a 5-4 vote. The court upheld the “substantial
likelihood of materially prejudicing that proceeding” test. Court struck down parts of rule that specified that an
attorney could make “general” statements about the defense “without elaboration” as unclear and vague. Court
upheld the “general” part of the test and struck down the “elaboration” part.

Rule 1.18: Duties to Prospective Client


● A person who discusses with a lawyer the possibility of forming a client-lawyer relationship
with respect to a matter is a prospective client.
● Even when no client-lawyer relationship ensues, a lawyer who has had discussions with a
prospective client shall not use or reveal information learned in the consultation.

Whether you can represent someone against a prospective client depends on whether you are in
possession of any confidential information of that person that can be used against them in the matter.

If the answer is yes, you can not represent the other side absent the prospective client’s consent.

The rule treats prospective clients like former clients: A lawyer that has had discussions with a
prospective client shall not use or reveal information learned in the consultation, except as Rule 1.9
would permit.

Rule 1.18 imposes an opportunity cost on consultations with prospective clients. The information you
receive from the prospective client may later disqualify you from representing the opponent of the
would-be-client.

Protect Yourself: Limit the information you receive at an initial consultation. Some attorneys only
accept “one side” of a case – workers comp example.

A consultation in which you learn nothing “significantly harmful” to the prospective client will not
disqualify you from later representing the other side in the same or a related matter. (Rest. Of the Law
Governing Lawyers, sec. 15(2),3rd Ed., 2000).

Prospective Client Hypo:


Potential new client “A” consults with you about a legal problem but doesn’t hire you.
Later, the adversary party “B,” meets with you, and requests your representation in the matter involving
“A.” Can you take the case? After all, you never represented “A.” Like it or not, you probably
learned something about the case from A. And, while you may not remember the consultation, A is
bound to recall it.

A True Story. Rose believed that she had a discrimination claim against GM. Rose consulted with an
attorney who was already representing other GM employees in identical claims. Misuse of
Confidential Information: The attorney declined the case and then tricked Rose into giving a
deposition on behalf of his other clients to support their claims. After GM obtained the deposition
testimony, Rose suffered retaliation in the workplace.

Example: Disqualifying technique in divorce cases.

Rule 1.17: Sale of Law Practice


● A lawyer (or firm) may sell or purchase a law practice (or an area of a law practice) including
good will.
● The seller must cease to engage in the prractic of law/area of practice, in the geographic area
● The seller must give wirtten notice to each of seller's clients regarding the proposed sale; the
client's right to retain other counsel/take possession of the file; AND the fact that client's
consent to transfer file will be presumed if client doesn't take action or object within 90 days of
the notice.
● If client cannot be contactd, court order must authorize transfer of file to the purchaser. Seller
may only disclose info necessary to complete transfer to judge in camera.
● Fees charged clients shall not be increased by reason of the sale.

Rule 3.2: Expediting Litigation


● A lawyer shall make reasonable efforts to expedite litigation consistent with the interests of the
client.

Be timely, if you wait until the last minute, bad things can happen. Malpractice, justice delayed = justice denied

The lawyer can be sanctioned.

If a reasonably compentent lawyer acting in good faither regards the course of action as having some substantial
purpose other than delay then it's ok. You cannot delay just so you client can take some financial advatange or
other benefit.

Rule 5.5: Unauthorized Practice Of Law; Multijurisdictional Practice Of Law


● A lawyer shall not practice law in a jurisdiction in violation of the regulation of the legal
profession in that jurisdiction, or assist another in doing so.
1. A lawyer who is not admitted to practice in this jurisdiction shall not:
2. establish an office
3. hold out to the public or otherwise represent that the lawyer is admitted to practice law
in this jurisdiction.
● A lawyer admitted in another jurisdiction (and not disbarred or suspended) may provide legal
services on a temporary basis in this jurisdiction that:
(1) are undertaken in association with a lawyer who is admitted in this jurisdiction OR
(2) are in or reasonably related to a pending or potential proceeding if the lawyer is
authorized by law or order to appear in such proceeding or reasonably expects to be so
authorized; [pro hac vice]
(3) are provided to the lawyer’s employer or its organizational affiliates and are not services
for which the forum requires pro hac vice admission; [corporate lawyers, govt lawyers]

What is “The Practice of Law?”


The practice of law consists of "the application of legal principles and judgment with regards to the
circumstances or objectives of a person that requires the knowledge and skill of a person trained in the
law." As a corollary, "a person is presumed to be practicing law when engaging in any of the following
conduct on behalf of another: Giving advice or counsel to persons as to their legal rights or
responsibilities or to those of others; Selecting, drafting, or completing legal documents or agreements
that affect the legal rights of a person; Representing a person before an adjudicative body, including but
not limited to, preparing or filing documents or conducting discovery; or Negotiating legal rights or
responsibilities on behalf of a person." ABA Task Force on the Model Definition of the Practice of
Law, Proposed Model Definition of the Practice of Law, Sections (b)(1) and (c) (Sept. 18, 2002).
Crossing State Lines (MJP)
A lawyer in State #1 may practice anywhere within State #1.

In order to appear in court outside State #1, the lawyer must be admitted to practice in that court either
through regular or special (pro hac vice) admission procedures.

A lawyer admitted to practice in State #1 may practice within a jurisdiction which the lawyer is not
admitted to the extent that the lawyer’s activities in the matter arise out of or are otherwise reasonable
related to the lawyer’s home practice.

Note: A lawyer can not appear in court in another jurisdiction unless admitted to practice in that court.
Tip: Does it “relate” all the way back to your practice in your home state? In other words, is it “within
the six degrees?” If so, it’s okay.

Rule 5.4: Professional Indepdence of a Lawyer


● A lawyer/firm shall not share legal fees w/a non-lawywer EXCEPT:
1. Agreement for payment of money over a reasonable period of time after a lawyer dies to
his estate or designated beneficiaries;
2. A lawyer who buys out share of a decesead, disabled, disappeared lawyer may pay to
estate or rep. an agreed upon purchase price.
3. firm may include nonlawyer employees in a compensation/retirement plan even though
the plan is based (in whole or in part) on a profit-sharing arrangement.
4. May share court-awarded legal fees with nonprofit that
employed/retained/recommended employment of the lawyer in the matter.
● A lawyer shall not form a partnership w/nonlawyer to practice law.
● A lawyer shall not permit anyone to direct/regulate the lawyer's professional judgement.
● A lawyer shall not practice law in a corp. IF a nonlawyer owns any interest therein (except as
fiduciary rep of estate of a lawyer);a nonlawyyer is corporate direcot or officer; OR nonlaywer
has the right to direct/control the professional judgment of a lawyer.

A lawyer can accept payment from third party if it doesn't interfere w/his professional judgement and
the client gives informed consent.

Rule 8.2: Judicial And Legal Officials


● A lawyer shall not make a statement that the lawyer knows to be false or with reckless disregard
as to its truth or falsity concerning the qualifications or integrity of a judge, adjudicatory officer
or public legal officer, or of a candidate for election or appointment to judicial or legal office.
● A lawyer who is a candidate for judicial office shall comply with the applicable provisions of
the Code of Judicial Conduct.

Rule 3.7: Lawyer As Witness


GR: You shall not serve as a client’s advocate in a matter in which you are likely to be a necessary
witness.
Exceptions
1. The testimony relates to an uncontested issue;
2. The testimony relates to the terms and value of the attorney fee; or,
3. Disqualification would work substantial hardship on the client.

Query
If you know that you are likely to be a necessary witness in a matter, can another lawyer with whom
you are associated act as the client’s advocate?
Rule 3.7(b): A lawyer may act as an advocate in a trial in which another lawyer in the lawyer’s firm is
likely to be called as a witness unless precluded from doing so by Rule 1.7 or Rule 1.9 (an improper
conflict of interest).
Example – it would be improper where there is likely to be substantial conflict between the
testimony of the client and that of the lawyer/witness.

When Might a Lawyer be Needed as a Witness?


Martha Stewart was charged with having given false statements to the government about the sale of her
Imclone stock. At trial, Stewart called as a witness the lawyer that represented her in the pre-
indictment phases of the case.

2005 Pistons/Pacers Basketbrawl: John Green, allegedly threw a cup of liquid at Ron Artest sparking
the infamous basketbrawl. Green was identified from a videotape by his lawyer. Green then denied his
involvement and the prosecutor named the lawyer as a necessary witness.

Business dispute over the structure of a deal or partnership that the lawyer was integrally involved in.

Will contest regarding the testator’s intent where lawyer drafted the will.

Problems when lawyer serves as both advocate and witness for a client:
● Confuses the jury.
● Undue prejudice to the adversary.
● Danger of prejudice to your own client if you are not deemed a credible witness.

Rule 2.1: Advisor


● In representing a client, a lawyer shall exercise independent professional judgment and render
candid advice. In rendering advice, a lawyer may refer not only to law but to other
considerations such as moral, economic, social and political factors, that may be relevant to the
client's situation.

Must tell them that there is no legal basis for a lawsuit.

Might want to tell them that filing the lawsuit would not be good for their careers and could cause bad
PR for everyone involved.

Rule 7.5: Firm Names And Letterheads


● A lawyer may use any firm name that is not false or misleading including a trade name as long
as it does not imply a connection with a government agency or public or charitable legal
services organization.
● A firm with offices in multiple jurisdictions can use the same name in all jurisdictions as long
as the lawyers working at each office are clearly identified along with any jurisdictional
limitations on their ability to practice.
● A lawyer holding public office may not be included in the name of the firm during any period in
which the lawyer is not actively and regularly practicing with the firm.
● Lawyers may only state or imply that they practice in a partnership or other organization when
this is a fact.

G.A. v Dennis Moffett (1985) Moffett practiced under two identities: Dennis Moffett, P.C. (a general
practice firm) and “The Malpractice Group, Inc.” which used as its slogan, “We only sue lawyers.”
Moffett was the sole lawyer in both enterprises. Any problem?

lawyers sharing office space, but not otherwise associated may not call themselves “Jones and Smith”
for example as if they are practicing law together in a firm.

It is misleading to use the name of a lawyer not associated with the firm or a predecessor of the firm, or
name of non-lawyer.

Can use deceased partner's name (it is considered a trade name).

Rule 1.12: Former Judge, Arbitrator, Mediator, or Other Third-Party Neutral


● Except as stated in below, a lawyer shall not represent anyone in connection with a matter in
which the lawyer participated personally and substantially as a judge or other adjudicative
officer or law clerk to such person or as an arbitrator, mediator or other third party neutral,
unless all parties give informed consent, confirmed in writing.
Example: Lawyer that sat as a juror (adjudicator) can not thereafter serve as counsel for
one of the parties on appeal absent consent.

● A judge (or adjudicative officer, abitrator, mediator or 3rd party neutral) shall not negotiate for
employment with any person who is involved as a party or as a lawyer for a party in a matter in
which the judge is presiding over.
● A law clerk may negotiate for employment with a party or lawyer for a party in a matter in
which the clerk is participating in but only after notifying the judge or other adjudicative
officer.
● If a lawyer is disqualified, no lawyer in a firm with which that lawyer is associated may
knowingly undertake or continue representation in the matter, unless:
1) the disqualified lawyer is timely screened from any part in the matter and receives no
part of the fee there from; and
2) written notice is promptly given to the parties and the appropriate tribunal to enable
them to ascertain compliance with this rule.
● An arbitrator selected as a partisan of a party in a multi-member arbitration panel is not
prohibited from subsequently representing that party.

This Rule overns conflicts involving former judges, arbitrators, mediators and judicial officers.

Rule 4.4: Respect For Rights Of Third Persons


● In representing a client, a lawyer shall not use means that have no substantial purpose other than
to embarrass, delay, or burden a third person, or use methods of obtaining evidence that violate
the legal rights of such a person.
● A lawyer who receives a document relating to the representation of the lawyer's client and
knows or reasonably should know that the document was inadvertently sent shall promptly
notify the sender.

Example: don't subpeona a person whom you don't plan to call just to harrass them

Example: In 2004, a California lawyer accidentally left a document in his conference room. Opposing
counsel, a 20 year veteran lawyer, discovered it while the other attorney was in the rest room. Rather
than ignoring it or turning it over, he illicitly made copies of it and then used the content to later
impeach the adversary’s witness. Holding: Offending lawyer was disqualified from the case.

Holland v Gordy Co. (MI 2004):


Defendants intentionally provided plaintiff with 12 boxes of documents pursuant to a document
production request. Defendants inadvertently included a 13th box which contained confidential
litigation files (work product). Plaintiff failed to notify defendant of the inadvertently produced
documents and did not return them. Holding: Plaintiff’s counsel committed ethical violations in
reviewing defendant’s litigation files and in failing to notify defendant about the production.

Rule 7.4: Communication of Fields of Practice and Specialization


● A lawyer may communicate the fact that the lawyer does/does not practice in particular fields of
law.
● A lawyer admitted to engage in patent practice before the United States Patent and Trademark
Office may use the designation "Patent Attorney" or a substantially similar designation.
● A lawyer engaged in Admiralty practice may use the designation "Admiralty," "Proctor in
Admiralty" or a substantially similar designation.
● A lawyer shall not state or imply that a lawyer is certified as a specialist in a particular field of
law, unless:
(1) the lawyer has been certified as a specialist by an organization that has been
approved by an appropriate state authority or that has been accredited by the American
Bar Association; and
(2) the name of the certifying organization is clearly identified in the communication.

7.4 is a codification of the Peel case and permits lawyers to advertise their fields of practice.
Peel v Attorney Registration & Disciplinary Comm. of Ill. (1990)
Facts: The attorney’s letterhead truthfully stated that he was a civil trial specialist as certified by the
National Board of Trial Advocacy (NBTA), a bonafide private group with objective and demanding
standards and procedures for lawyer certification.
Illinois had an ethics rule that prohibited advertising as “certified” or as a “specialist” unless a patent,
trademark or copyright attorney.
Held: The U.S. Supreme Court reversed finding that the ad was truthful and neither actually nor
inherently potentially misleading.

Prohibits certification as a specialist unless you are actually certified by an approved and recognized
certifying organization and the organization is listed in the ad.

Although you may generally represent that you are a “specialist” or that you “specialize” in an area of
practice, such representations are subject to the false and misleading standard of Rule 7.1.
Rule 6.2: Accepting Appointments
A lawyer cannot avoid court appointments except for good cause such as:
● rep will likely result in violation of the Rules or other law
● rep results in unreasonable financial burden (happens in solo practice + huge capital case like OK city
bomber)
● client/cause so repugnant to lawyer so as to impair attny client relationship or lawyer's ability to
represent the client. (Although this is good cause, this will get you kicked off court appointed list very
quickly)

Not good cause:


1. cause is unpopular
2. influential members of community or other clients oppose involvement
3. You believe the person is guilty (unless your personal feelings are so intense that effective
representation is impaired)

GR: “lawyer is not a public utility.” That means that a lawyer has no duty to serve just anybody who
wants service and can pay for it.

Example Problem
But what about the Oath to represent the cause of the defenseless or the oppressed?
Nazi party has ample money to pay.
Other skilled lawyers have refused to serve as counsel.
Clearly, Nazi party is unpopular client.

What if she is appointed?


She may refuse the appointment if taking the case would violated a disciplinary rule.
- For example, if she was the only eyewitness to a key contested fact in the case. (Rule 3.7)

She may refuse if by taking the case it would cause an unreasonable financial burden on her.
Rule 6.2(b). For example, she may be a solo practitioner and the trial is scheduled to take
months to complete.

She may refuse if her personal feelings make the cause so repugnant so to impair the attorney-
client relationship or her ability to represent the client. Rule 6.2(c). She is Jewish and her
grandmother narrowly escaped from Austria in 1939. Need to know more about her personal
feelings.

Earl case
In 2004, Washington public defender Thomas J. Earl was disbarred for repeatedly soliciting and
accepting fee payments from court-appointed clients he was supposed to represent for free.
At the related disciplinary hearing the attorney for the bar picked up a green marker and drew a dollar sign. "Mr.
Earl put a price tag on the administration of justice."

Who is entitled to court appointed counsel?


Gideon v. Wainwright, 372 U.S. 335 (1963) Held: The right of an indigent defendant in a criminal
case (capital or non-capital) to have the assistance of counsel is a fundamental right essential to a fair
trial. The assistance of counsel is one of the safeguards of the Sixth Amendment deemed necessary to
insure fundamental human rights of life and liberty...Trial and conviction without the assistance of
counsel violates the Fourteenth Amendment.

Are Appointments Ever made in Civil Cases?


Bothwell v Republic Tobacco (1995) Facts: Plaintiff was incarcerated and filed a civil complaint and a
motion for appointment of counsel. Plaintiff made a products liability claim. Judge appointed civil
counsel who then moved for reconsideration of appointment.
Issue: Does a federal court possess the inherent power to compel and unwilling attorney to accept a
civil appointment?
Holding: Yes. Even though the court did not find it necessary to exercise its inherent authority in this
case, enough policy reasons and theories existed to establish that federal courts had the power to
compel and unwilling attorney to represent an indigent client in a civil matter.
Factors:
✔ Factual and legal complexity of the claims.
✔ The plaintiff’s ability to investigate the facts.
✔ The existence of conflicting testimony.
✔ The plaintiff’s ability to present his claims.
✔ The plaintiff’s ability to obtain counsel on his own. (Marketability Analysis).
✔ Marketability Analysis:
✔ Was there a market of lawyers who practice in the area of plaintiff’s claims?
✔ Did the plaintiff have ready access to that market?
✔ What are the possible fee arrangements for these types of claims?
✔ Was the market’s rejection of plaintiff’s claims due to indigency which triggers the
court’s ability to compel representation?

Rule 2.3: Evaluation for Use by third Persons


● A lawyer may provide an eval of matter affecting client if lawyer reasonably believes providing
the eval is compatible with repping client.
● If the eval is likely to affect client's interest adversely and materially the lawywer cannot
provide the eval UNLESS the clinet ives informed consent.
● Except disclosure as auth'd, info relating to eval is otherwise protected by 1.6.

legal duty to third peron may/may not arise.

Rule 6.1: Indigent Representation


● Every lawyer is professionally obligated to provide legal services to those unable to pay. You
should aspire to at least 50 hours of pro bono work per year.

This rule is not compulsory. It is aspirational only. (Forced representation without compensation
raises constitutional issues). Remember: Excerpt From The Lawyer’s Oath: “I will never reject, from
any consideration personal to myself, the cause of the defenseless or oppressed…”

Rule 5.7: Responsibilities Re: Law Related Services


● A lawyer is subject to the rules if he provides law related services in circumstances that aren't
distinct from the lawyer's legal servics to clients OR if the lawyer fails to take reasonable
measure to make sure that persons understand that law-related services aren't the same as legal
services (and the protections of attny-client priv don't apply).

The burden is on the lawyer to show he took reasonable measures.

Confusion happens when lawyer provides law related services and regular legal services.

Examples of law related services: insurance, financial planning, accounting, trust services, real estate
consulting, tax preparation, medial or environmental consulting.

Rule 4.3: Dealing With Unrepresented Persons


● In dealing on behalf of a client with a person who is not represented by counsel, you shall not
state or imply that you are disinterested.
● If you know or have reason to know that a person misunderstands your role, you must take
reasonable efforts to correct the misunderstanding.
● You shall not give legal advice to an unrepresented person, other than advice to secure counsel,
if you know or reasonably should know that the interests of such a person are or have a
reasonable possibility of being in conflict with the interests of the client.

Not on any tests

Rule 3.9: Advocate In Nonadjudicative Proceedings


● A lawyer representing a client before a legislative body or administrative agency in a
nonadjudicative proceeding shall disclose that the appearance is in a representative capacity and
shall conform to the provisions of the Rules.
● Lay people can often represent persons in legislative or administrative proceedings such as
workers’ compensation matters and social security claims.
● Rule 3.9 requires that if a lawyer is representing the claimant, the Rules of Professional
Conduct apply to the lawyer’s conduct in the proceedings.
● Lawyers have a duty to advise such agencies that they are present in a lawyer capacity.

Rule 1.10: Imputation


● While associated in a firm no member of the firm shall knowingly represent a client when any
one member practicing alone would be prohibited from doing so under 1.7 or 1.9, unless:
1) the prohibition is based on a personal interest; and,
2) it doesn’t present a significant risk of materially limiting the representation of the
client by someone else in the firm.
● prevents a former firm (absent informed consent/confirmed in writing) from taking cases
against clients that left with a departing lawyer.
● A disqualification prescribed by this Rule may be waived by the affected client under the
conditions stated in Rule 1.7(b).
Example: A waiver is permissible as long as the conflict is not so direct and intense that
the representation is per se impermissible.

LAXTON: typically conflicts ARE waivable but in practice it doesn't happen b/c of the potential for
ethical problems amicable divorce w/ no children....lawyer represents both husband and wife and puts
together the paperwork. No negotiation...in very limited circumstances can you do that.
Government Lawyers: Rule 1.10(d) sets forth that the disqualification of lawyers associated in a firm
with former or current government lawyers is governed by the “screening” provisions in Rule 1.11(b).
This rule codifies the principal of loyalty as applied to a law firm.

Pursuant to Rule 1.10, a firm of lawyers is essentially one lawyer for purposes of the conflict rules.

Rule 1.10 does not apply to conflicts of non-lawyer employees of the firm, including law clerks. These
persons, however, if in conflict, must be screened off.

Rule 8.5: Disciplinary Authority


● A lawyer admitted to practice in this jurisdiction is subject to the disciplinary authority of this
jurisdiction, regardless of where the lawyer's conduct occurs. A lawyer not admitted in this
jurisdiction is also subject to the disciplinary authority of this jurisdiction if the lawyer provides
or offers to provide any legal services in this jurisdiction. A lawyer may be subject to the
disciplinary authority of both this jurisdiction and another jurisdiction for the same conduct.
● For conduct in connection with a matter pending before a tribunal, the rules of the jurisdiction
in which the tribunal sits, applies (unless the rules of the tribunal provide otherwise) AND
● For any other conduct, the rules of the jurisdiction in which the lawyer’s conduct occurred, or,
if the predominant effect of the conduct is in a different jurisdiction, the rules of that
jurisdiction shall be applied to the conduct. A lawyer shall not be subject to discipline if the
lawyer’s conduct conforms to the rules of a jurisdiction in which the lawyer reasonably believes
the predominant effect of the lawyer’s conduct will occur.

The Departing Lawyer


A departing lawyer may solicit firm clients only on matters the lawyer worked actively and
substantially and only after the lawyer has left the firm or after the lawyer has adequately and timely
informed the firm of an intent to contact clients for that purpose. Rest. Of the Law Governing
Lawyers, 3rd, § (3).

Rationale: Clients are not merchandise. They have the right to counsel of their choice.
Best Practice: Joint letter to clients from departing lawyer and the firm giving clients their choice of
counsel.

Elements of a Legal Malpractice Claim


Duty
You must owe a duty for there to be a breach of duty. An attorney-client relationship creates a
duty.
Sometimes, a duty is present in the absence of an attorney-client relationship.
Examples Services for the benefit of third persons. (E.g., wills and third party
evaluations or opinion letters). confidences of a prospective client (Rule 1.18)

Breach
The standard of care is generally: the skill and knowledge ordinarily possessed by attorneys
under similar circumstances.
Not liable for mere errors in judgment – but must be well informed.
Trial strategy vs Failure to Investigate

Actual Cause
TEST: But-for the act or omission complained of, the client would have received a better
result.
Or Substantial Factor Test when multiple causes of injury are present.
The “case-within-the-case” requirement. P must show by a preponderance what the attorney
should have done and that it would have made a difference and D counters by showing that it
was a harmless error and the outcome would not have been different.
Example:
Criminal defendant pleads guilty and is sentenced to incarceration. Defendant brings
legal malpractice action blaming Lawyer for the outcome of his case. Defendant’s claim
will fail because Defendant can not show that but-for the Lawyer’s action/inaction,
Defendant would have had a different result. Defendant’s conviction is a direct result of
Defendant’s plea of guilty not anything Lawyer did/did not do.

Ineffective Assistance of Counsel is NOT the same as legal malpractice

Proximate Cause

Damages
Direct vs Consequential
Anticipated and Natural vs Indirect but Foreseeable
Cannot recover for legal malpractice absent evidence of actual damages.
Damages must be pled and proven.

Vicarious Liability: Law firms are vicariously liable for the legal malpractice of partners and
associates.

Malpractice Insurance: Not mandatory under the ABA code. Oregon is only state that
requires coverage. According to the ABA, only 30 to 40% of attorneys carry malpractice
insurance. It is very, very expensive but not as expensive as a valid claim – to both your wallet
and your reputation!

Arbitration Agreements:
Agreements to arbitrate malpractice claims are ethical as long as:
1. the agreement does not insulate the lawyer from liability; and,
2. the client understands what it means.

Analysis Tools
Things to Consider in Deciding Whether to Accept a Case:
Type of Matter.
1. Do you have the competency?
(Rule 1.1).
2. Do you have the time? (diligence)
(Rule 1.3).
3. Does the matter have merit?
(Rule 3.1).
The Client.
Advertising v. Direct Solicitation
ADVERTISING: Communication with the public at large.
DIRECT SOLICITATION: Involves personal contact. Direct Solicitation is Prohibited!
Why? The Dangers:
Fraud.
Undue influence.
Overreaching.
Diminished respect for the legal profession.

The Problem:
Direct Solicitation involves:
1. a sales pitch,
2. via direct contact,
3. to an already vulnerable person, and;
4. pressure for an immediate response.

Prohibited Direct Contact:


In person
Live telephone
Real-time electronic

Вам также может понравиться